Urden Questions #3

Lakukan tugas rumah & ujian kamu dengan baik sekarang menggunakan Quizwiz!

A patient is admitted with a massive head trauma. The patient is unresponsive and on mechanical ventilation. The electroencephalogram (EEG) is negative for brain waves. The family has agreed to organ donation. How would this organ donor be classified? a. Brain-dead donor b. Donation after cardiac death c. Living related donor d. Living unrelated donor

A A brain-dead donor is a donor declared dead by neurologic criteria for brain death. Donation after cardiac death is when the donor is declared dead by circulatory criteria for death. A living related donor is a family member related by blood who donates a kidney, portion of the liver, pancreas, intestine, or lung to another family member. A living unrelated donor (directed or nondirected) is a living individual not related to a patient requiring a transplant who donates a kidney, portion of the liver, pancreas, intestine, or lung to that patient. The donor may be anonymous or altruistic.

A patient who has received a transplant is being taught about azathioprine. Which statement made by the patient would indicate the teaching was effective? a. "I will notify my health care provider if my gums start to bleed." b. "I will make sure to increase the amount of fiber in my diet." c. "I realize I may have an increase in hair growth." d. "I know the flulike symptoms will go away as I get used to the drug."

A A common adverse effect is the suppression of other rapidly proliferating cells, resulting in leukopenia, thrombocytopenia, and anemia. The dose of the drug is adjusted to keep the white blood cell (WBC) count between 3000 and 5000 cells/mm3, thus protecting the patient from an increased risk of infection.

A patient with acute kidney injury (AKI) has been started on continuous venovenous hemodialysis (CVVHD). The nurse knows the hemodialyzer filter used in this type of therapy is permeable to what substance? a. Electrolytes b. Red blood cells c. Protein d. Lipids

A A continuous venovenous hemodialysis filter is permeable to solutes such as urea, creatinine, uric acid, sodium, potassium, ionized calcium, and drugs not bound by proteins.

The nurse is caring for a patient who has been newly diagnosed with type 1 diabetes. The nurse notes that the patient is extremely dehydrated. To make this assessment, what did the nurse evaluate? a. Skin turgor b. Nail bed color c. Capillary refill d. Skin temperature

A A hydration assessment includes observations of skin integrity, skin turgor, and buccal membrane moisture. Moist, shiny buccal membranes indicate satisfactory fluid balance. Skin turgor that is resilient and returns to its original position in less than 3 seconds after being pinched or lifted indicates adequate skin elasticity. Skin over the forehead, clavicle, and sternum is the most reliable for testing tissue turgor because it is less affected by aging and thus more easily assessed for changes related to fluid balance.

A nurse is teaching a patient with diabetes mellitus. The patient asks the nurse what is an acceptable HbA1c level for him. What should the nurse tell the patient? a. An acceptable level is less than 5.4%. b. An acceptable level is less than 6.5%. c. An acceptable level is determined by your practitioner. d. It is dependent on your age.

A A normal HbA1C value is less than 5.4%, with an acceptable target level for patients with diabetes below 6.5%.

When renin eventually stimulates angiotensin II, the adrenal glands then secrete what hormone? a. Aldosterone b. Adrenal stimulating hormone c. Antidiuretic hormone d. Vasopressin

A A reduction in vascular volume stimulates the release of renin. Renin converts to angiotensin I, which converts to the powerful vasoconstrictor angiotensin II. In turn, angiotensin II stimulates the adrenal glands to secrete aldosterone, which acts on the distal tubules to resorb sodium from the tubular lumen into the circulation.

Which diuretics maybe combined to work on different parts of the nephron? a. Loop and thiazide diuretics b. Loop and osmotic diuretics c. Osmotic and carbonic anhydrase inhibitor diuretics d. Thiazide and osmotic diuretics

A A thiazide diuretic such as chlorothiazide (Diuril) or metolazone (Zaroxolyn) may be administered and followed by a loop diuretic to take advantage of the fact that these medications work on different parts of the nephron. Sometimes a thiazide diuretic is added to a loop diuretic to compensate for the development of loop diuretic resistance.

A patient with acute kidney injury has a potassium level of 6.9 mg/dL. The patient has had no urine output in the past 4 hours despite administration of Lasix 40 mg intravenous push. To correct the hyperkalemia the patient is given 50 mL of 50% dextrose in water and 10 U of regular insulin intravenous push. A repeat potassium level 2 hours later shows a potassium level of 4.5 mg/dL. What order would the nurse expect now? a. Sodium Kayexalate 15 g PO b. Nothing; this represents a normal potassium level c. Lasix 40 mg IVP d. 0.9% normal saline at 125 mL/h

A Acute hyperkalemia can be treated temporarily by intravenous (IV) administration of insulin and glucose. An infusion of 50 mL of 50% dextrose accompanied by 10 units of regular insulin forces potassium out of the serum and into the cells. However, the potassium was not eliminated from the body; it was simply shifted intracellularly. Soon the potassium will return to the bloodstream, and the Kayexalate will help permanently remove it from the body. Lasix is not expected to work in the presence of anuria. The patient's vital signs do not support hypovolemia. In the presence of anuria, a large fluid infusion can precipitate acute heart failure.

A patient has had a kidney transplant. The nurse knows that monitoring of which parameter would have the highest priority? a. Fluid volume b. Electrolytes c. Complete blood count d. Temperature

A Adequate hydration is an absolute necessity for continued graft function in the immediate postoperative period. Hypovolemia can lead to compromised blood flow to the kidney, acute tubular necrosis, and possible graft failure. The new kidney will produce large amounts of urine, and replacement fluids, usually maintained at a ratio of 1:1 mL, must be sustained.

What hormone do the alpha cells of the pancreas secrete? a. Glucagon b. Insulin c. Somatostatin d. Pancreatic polypeptide hormone

A Alpha cells secrete glucagon; beta cells secrete insulin; delta cells secrete somatostatin, and pancreatic polypeptide cells secrete pancreatic polypeptide hormone.

The nurse is caring for a patient after a heart transplant. Which finding would the nurse anticipate after cardiac transplantation? a. Two P waves on the electrocardiogram (ECG) b. High cardiac output c. Anginal pain d. Resting heart rate of 60 to 70 beats/min

A An electrocardiogram (ECG) abnormality unique to a transplanted heart is the presence of a second P wave generated by the native sinoatrial (SA) node left in the atrial cuff. Because this impulse does not cross the suture line, it is capable of conducting only through the remnant of the native recipient atria. However, this is not seen in hearts transplanted using the bicaval technique because the native right atrium and therefore the SA node is removed.

After a pancreatic transplant with bladder exocrine drainage, the nurse would anticipate which order? a. Continuous bladder irrigation b. Intermittent insulin injections c. Removal of the nasogastric tube as soon as the patient is alert d. Daily hematocrit and hemoglobin levels

A An increased potential for urinary catheter occlusion exists for pancreas recipients who have undergone a urinary diversion procedure. The exocrine pancreatic enzymes make the urine more viscous, and they irritate the anastomosis site on the bladder, causing an increased risk of bleeding. Continuous bladder irrigation may be necessary to keep the catheter patent.

A patient with thyrotoxicosis has been admitted. What effect of increased T3 and T4 levels might the nurse expect to see in this patient? a. Increased oxygen consumption b. Decreased basal metabolic rate c. Decreased cardiac output d. Increased calcium levels

A Both thyroid hormones impact the rate at which oxygen is used in the body and thus affect all metabolic processes in the body.

Loss of albumin from the vascular space may result in which condition? a. Peripheral edema b. Extra heart sounds c. Hypertension d. Hyponatremia

A Decreased albumin levels in the vascular space result in a plasma-to-interstitium fluid shift, creating peripheral edema. A decreased albumin level can occur as a result of protein-calorie malnutrition, which occurs in many critically ill patients in whom available stores of albumin are depleted. A decrease in the plasma oncotic pressure results, and fluid shifts from the vascular space to the interstitial space.

Decreased insulin and increased glucagon result in what physiologic process? a. Increased glycogenolysis b. Decreased fat mobilization c. Decreased ketosis d. Increased glycogen storage

A Decreased insulin and increased glucagon result in increased glycogenolysis.

Who determines the medical suitability of the patient for organ donation? a. The organ procurement organization (OPO) coordinator b. The patient's family c. The admitting health care provider d. The transplant team

A Determining medical suitability is solely the responsibility of the organ procurement organization (OPO). Speaking to the family about donation is also the responsibility of the OPO unless designated requestors at the hospital have been trained to do so.

A patient is admitted with diabetic ketoacidosis (DKA). The nurse knows that the dehydration associated with DKA results from which pathophysiologic mechanism? a. Increased serum osmolality and urea b. Decreased serum osmolality and hyperglycemia c. Ketones and potassium shifts d. Acute renal failure

A Hyperglycemia increases the plasma osmolality, and the blood becomes hyperosmolar. Cellular dehydration occurs as the hyperosmolar extracellular fluid draws the more dilute intracellular and interstitial fluid into the vascular space in an attempt to return the plasma osmolality to normal.

A patient with chronic kidney disease was admitted with severe electrolyte disturbances. The patient had been ill and missed several hemodialysis sessions. The patient is disoriented, dizzy, cold, clammy, and complains of severe abdominal cramping. The patient's electrocardiogram appears normal. Which electrolyte disturbance would the nurse suspect the patient may be experiencing? a. Hyponatremia b. Hypokalemia c. Hypercalcemia d. Hypochloremia

A Hyperkalemia, hypocalcemia, hyponatremia, hyperphosphatemia, and acid-base imbalances occur in kidney disease. Signs of hyponatremia include disorientation, muscle twitching, nausea, vomiting, abdominal cramps, headaches, dizziness, cold, clammy skin, tachycardia, and seizures.

A patient has been admitted who is suspected of having thyrotoxicosis. Which laboratory findings would confirm this diagnosis? a. Very low thyroid-stimulating hormone (TSH) b. Decreased T3 uptake ratio c. Increased serum osmolality d. Decreased urine osmolality

A Hyperthyroidism (thyrotoxicosis) is indicated by very low thyroid-stimulating hormone level, high serum T4, and an increased T3:T4 ratio.

Which nursing problem would be appropriate for a patient who has received a kidney transplant? a. Deficient fluid volume b. Excess fluid volume c. Urinary retention d. Decreased cardiac output

A Hypovolemia can lead to compromised blood flow to the kidney, acute kidney injury, and possible graft failure. The new kidney will be producing large amounts of urine, and fluid replacement, usually maintained in a 1:1 ratio, must be sustained.

A patient is presenting with signs of diabetes insipidus (DI). Which findings would confirm this diagnosis? a. Hyperosmolality and hypernatremia b. Hyperosmolality and hyponatremia c. Hypoosmolality and hypernatremia d. Hypoosmolality and hyponatremia

A In central diabetes insipidus (DI), there is an inability to secrete an adequate amount of antidiuretic hormone (arginine vasopressin) in response to an osmotic or nonosmotic stimuli, resulting in inappropriately dilute urine. Hypernatremia is usually associated with serum hyperosmolality.

The practitioner has ordered dialysis for a patient with acute heart failure who is unresponsive to diuretics. Which type of dialysis would the nurse anticipate being started on this patient? a. Intermittent ultrafiltration b. Continuous venovenous hemofiltration (CVVH) c. Continuous venovenous hemodialysis (CVVHD) d. Continuous venovenous hemodiafiltration (CVVHDF)

A Intermittent ultrafiltration using a peripheral venous catheter is more likely to be used to remove excess volume from patients with acute decompensated heart failure when the kidneys are unresponsive to diuretics.

A patient is admitted with acute kidney injury (AKI). Which event from the patient's history was the most probable cause of the patient's AKI? a. Recent computed tomography of the brain with and without contrast b. Recent bout of acute heart failure after an acute myocardial infarction c. Twice-daily prescription of Lasix 40 mg by mouth d. Recent bout of benign prostatic hypertrophy and transurethral resection of the prostate

A Intravenous contrast media can be nephrotoxic, especially with the patient's preexisting cardiac disease. The other choices, although possible causes, are less likely than the intravenous contrast media.

A patient presents with ketoacidosis and a blood glucose level of 125 mg/dL. Diabetic ketoacidosis has been ruled out. The nurse knows that ketoacidosis can occur in which condition? a. Acute pancreatitis b. Drug overdose c. Hyperglycemic hyperosmolar state d. Hyperaldosteronism

A Ketoacidosis also occurs with acute pancreatitis, starvation, and alcoholic ketoacidosis.

Which statement regarding kidney function and aging is accurate? a. Kidney function declines with age but this usually does not affect homeostasis. b. Serum creatinine rises with age due to increased catabolism of red muscle. c. Glomerular filtration rate falls at a rate of 2 mL/min/year. d. Older adults are less susceptible to acute kidney dysfunction but more susceptible to chronic kidney dysfunction.

A Kidney function declines gradually with age, but this usually does not affect homeostasis in the healthy older adult unless proteinuria is present. Proteinuria is associated with complication in both the kidney and cardiovascular systems. However, despite the gradual decrease in the glomerular filtration rate (GFR) and the associated reduction in clearance of creatinine, serum creatinine levels may not rise. With aging, the GFR declines by about 0.75 mL/min/year. When older adults become ill, the decline in kidney function can be accelerated, making older adults especially susceptible to acute and chronic kidney dysfunction.

A nurse is preparing a patient for a liver transplant. The patient asks which stage of the procedure is the longest and most difficult. What would the nurse tell the patient? a. Stage 1—recipient hepatectomy b. Stage 2—vascular anastomoses c. Stage 3—biliary anastomosis d. Stage 4—induction therapy

A Liver transplant surgery can be divided into three stages: (1) recipient hepatectomy (the longest and most difficult part of the surgery), (2) vascular anastomoses with donor liver, and (3) biliary anastomosis. Induction therapy, which refers to the first in any series of therapeutic measures taken to treat a disease, would be administered after the transplant and is not part of the procedure.

A patient has been admitted with acute kidney injury. The nurse knows the most important consideration for evaluating the patient's fluid status is what parameter? a. Daily weights b. Urine and serum osmolality c. Intake and output d. Hemoglobin and hematocrit levels

A One of the most important assessments of kidney and fluid status is the patient's weight. In the critical care unit, weight is monitored for each patient every day and is an important vital signs measurement.

What substance is the most responsible for maintaining the colloid osmotic pressure? a. Intravascular plasma proteins b. Intracellular potassium c. Extracellular sodium d. Interstitial potassium

A Osmotic pressure is created by solutes and other substances (eg, albumin, globulin, fibrinogen) suspended in fluid. Colloid osmotic pressure is created primarily by the presence of plasma proteins in the intravascular space. Plasma proteins exert a pull on water molecules and therefore produce osmotic pressure, which retains fluid within the intravascular compartment.

A nurse is caring for a patient who is receiving sirolimus. The nurse knows to monitor the patient for what side effect? a. Bleeding b. Diarrhea c. Rigors d. Hypotension

A Other primary side effects of this medication include hyperlipidemia and myelosuppression. Most of the myelosuppressive effect is directed at platelets, and severe thrombocytopenia can result, making it necessary to discontinue the medication.

The nurse is developing a discharge education plan for a patient with syndrome of inappropriate secretion of antidiuretic hormone (SIADH). Which topic should be included in the plan? a. Measuring intake and output b. Encouraging fluids c. A low-sodium diet d. Hypothermia management

A Patient and family education about syndrome of inappropriate secretion of antidiuretic hormone (SIADH), its effect on water balance, and the reasons for fluid restrictions using the teach back method should include how to measure intake and output.

The nurse is caring for a patient with a head injury who has developed diabetes insipidus (DI). What medication would the nurse expect to be prescribed for the patient? a. Vasopressin b. Insulin c. Glucagon d. Propylthiouracil

A Patients with central diabetes insipidus (DI) who are unable to synthesize antidiuretic hormone (ADH) require replacement ADH (vasopressin) or an ADH analog. The most commonly prescribed drug is the synthetic analog of ADH, desmopressin (DDAVP). DDAVP can be given intravenously, subcutaneously, or as a nasal spray. A typical DDAVP dose is 1 to 2 mcg intravenously or subcutaneously every 12 hours.

A patient is admitted with diabetic ketoacidosis (DKA). The patient presents with dry, cracked lips and is begging for something to drink. What reply would be the nurse's best response? a. "We can't give you anything to drink until we get your blood sugar under control." b. "You can have one cup of coffee without sugar." c. "You can drink anything you want as long as it's sugar free." d. "You can drink as much water has you can handle."

A Patients with diabetic ketoacidosis (DKA) are kept on NPO status (nothing by mouth) until the hyperglycemia is under control.

Which findings may be present in the patient with significant fluid volume overload? (Select all that apply.) a. S3 or S4 may develop. b. Distention of the hand veins will disappear if the hand is elevated. c. When testing the quality of skin turgor, the skin will not return to the normal position for several seconds. d. Tachycardia with hypotension may be present. e. Dependent edema may be present.

A, E A gallop and dependent edema are indicative of fluid excess; the other signs are indicative of fluid volume deficit.

The nurse is caring for a patient with a pancreas transplant. Which statement made by the patient indicates the need for further teaching? a. "I no longer need to monitor my blood glucose levels." b. "I will need to have periodic pancreas biopsies." c. "I may feel the urge to urinate frequently." d. "I will give urine samples to monitor for rejection."

A Patients with functional pancreas grafts continue to need glucose monitoring at home but often forget to continue this practice after they no longer require insulin. Continued monitoring with frequent clinic visits is required for several months after pancreas transplantation.

A patient has been admitted with diabetic ketoacidosis, and treatment has been initiated. Which findings would lead the nurse to suspect the patient is dehydrated? a. Poor skin turgor and flat neck veins b. Dyspnea and crackles c. Presence of Chvostek and Trousseau signs d. Extra heart sounds and 3+ edema

A Poor skin turgor and flat neck veins are indicative of dehydration. Dyspnea, crackles, extra heart sounds, and 3+ edema are signs of fluid overload. Chvostek and Trousseau signs are indicative of hypocalcemia.

The nurse is caring for a patient with type 1 diabetes who was admitted with complaints of increased lethargy. Serum laboratory values validate the diagnosis of diabetic ketoacidosis (DKA). Which medical intervention would the nurse expect to be ordered for this patient? a. Extensive hydration b. Oral hypoglycemic agents c. Large doses of intravenous (IV) insulin d. Limiting food and fluids

A Rapid IV fluid replacement requires the use of a volumetric pump. Insulin is administered intravenously to patients who are severely dehydrated or have poor peripheral perfusion to ensure effective absorption. Patients with diabetic ketoacidosis (DKA) are kept on NPO (nothing by mouth) status until the hyperglycemia is under control. Critical care nurses are responsible for monitoring the rate of plasma glucose decline in response to insulin.

A patient has been diagnosed with syndrome of inappropriate secretion of antidiuretic hormone (SIADH). The nurse would expect the first line treatment to include which intervention? a. Fluid restriction b. Hypotonic intravenous fluid administration c. Administration of D5W d. Administration of vasopressin

A Reduction in fluid intake is one component of the treatment plan for syndrome of inappropriate secretion of antidiuretic hormone (SIADH).

A patient has been admitted with hyperosmolar hyperglycemic state (HHS). The nurse knows that intravenous insulin is usually administered at what dosage? a. 0.1 U/kg/h b. 1.0 U/kg/h c. 2.0 U/kg/h d. 5.0 U/kg/h

A Regular insulin infusing at an initial rate calculated as 0.1 unit per kg hourly (7 U/h for a person weighing 70 kg) should lower the plasma glucose by 50 to 70 mg/dL in the first hour of treatment. If the measured glucose does not decrease by this amount, the insulin infusion rate may be doubled until the blood glucose is declining at a rate of 50 to 70 mg/dL/h.

As a patient with diabetic ketoacidosis (DKA) receives insulin and fluids, the nurse knows careful assessment must be given to which electrolyte? a. Potassium b. Sodium c. Phosphorus d. Calcium

A Replacement of potassium by administration of potassium chloride (KCl) begins as soon as the serum potassium falls below normal. Frequent verification of the serum potassium concentration is required for the patient with diabetic ketoacidosis receiving fluid resuscitation and insulin therapy.

A patient is being evaluated for thyroid dysfunction. The patient's medications include aspirin, digoxin, chlorothiazide, and insulin. The nurse knows that the laboratory work may be affected by which of these medications? a. Aspirin b. Digoxin c. Insulin d. Chlorothiazide

A Several drugs increase the serum level of T4 by displacing protein-bound T4. Drugs that displace T4, including heparin (both unfractionated and low-molecular-weight heparins), cause an increase in serum T4 levels. Salicylates (aspirin) and furosemide (Lasix) also raise T4 serum levels by the same mechanism.

A patient is diagnosed with a pheochromocytoma. What signs or symptoms might the nurse expect to see in this patient? a. Hypertension and tachycardia b. Weight gain and acne c. Diarrhea and facial swelling d. Blurred vision and poor skin turgor

A Signs and symptoms of pheochromocytoma include hypertension and tachycardia. Weight gain and acne are signs of Cushing syndrome. Diarrhea and facial swelling are signs of hyperthyroidism. Blurred vision and poor skin turgor are signs of hyperglycemia.

A patient was admitted with asthma who has been on steroids for a long time. The nurse knows that this patient is at risk of developing secondary Cushing syndrome. What would be the cause of this condition? a. Pharmacologic glucocorticoids b. Pharmacologic mineralocorticoids c. Cortisol d. Aldosterone

A Symptoms identical to those of primary Cushing syndrome occur in patients with the secondary form who chronically take pharmacologic doses of glucocorticoids, for example, transplant recipients who take steroids to prevent solid organ rejection, patients with chronic obstructive pulmonary disease, or those with chronic inflammatory conditions. Cortisol and aldosterone are hormones released by the adrenal gland.

The Model for End-Stage Liver Disease (MELD) formula is used to calculate risk of 3-month mortality in patients 12 years old or older. What criteria is part of the MELD formula? a. Whether the patient has undergone hemodialysis at least twice in the past 2 weeks b. The number of organs in which metastasis has occurred following hepatocellular carcinoma c. The presence or absence of intractable pruritus d. The number of hours the patient is expected to live without a transplant

A The Model for End-Stage Liver Disease (MELD) formula is used in all U.S. transplant centers to calculate the risk of 3-month mortality in patients 12 years old or older. The MELD objective criteria include serum total bilirubin, serum creatinine, prothrombin time, international normalized ratio, and whether the patient has undergone hemodialysis at least twice in the past 2 weeks. Patients who have hepatocellular carcinoma that meet specific tumor criteria are automatically given a MELD score of 22 because the risk of metastasis outside the liver within 3 months is high. Once metastasis occurs, the patient is no longer deemed a transplant candidate. Intractable pruritus and number of hours to live are not part of the MELD formula.

When calculating the anion gap, what is the predominant cation? a. Sodium b. Potassium c. Chloride d. Bicarbonate

A The anion gap is a calculation of the difference between the measurable extracellular plasma cations (sodium and potassium) and the measurable anions (chloride and bicarbonate). In plasma, sodium is the predominant cation, and chloride is the predominant anion.

A patient is admitted with diabetic ketoacidosis. The nurse requests the practitioner order a glycosylated hemoglobin (HbA1C). What information does this test provide to the health care team? a. It is an indicator of the patient's average blood glucose level over the previous 3 to 4 months. b. It compares blood glucose levels with serum hemoglobin over the previous 3 to 4 weeks. c. It is an indicator of the patient's highest blood ketone level over the past month. d. It associates the serum and urine glucose levels and is an indicator of kidney involvement.

A The glycated hemoglobin test (also known as the glycosylated hemoglobin, or HbA1C or A1C) provides information about the average amount of glucose that has been present in the patient's bloodstream over the previous 3 to 4 months. During the 120-day life span of red blood cells (erythrocytes), the hemoglobin within each cell binds to the available blood glucose through a process known as glycosylation.

The nurse has admitted a patient with hyperglycemic hyperosmolar state (HHS). Which medical intervention would the nurse expect to see ordered for this patient? a. Rapid rehydration with intravenous fluids b. Insertion of a pulmonary artery catheter c. Administration of high-dose intravenous insulin d. Hourly monitoring of urine glucose and ketone levels

A The goals of medical management are rapid rehydration, insulin replacement, and correction of electrolyte abnormalities, specifically potassium replacement. The underlying stimulus of the hyperglycemic hyperosmolar state (HHS) must be discovered and treated. The same basic principles used to treat patients with diabetic ketoacidosis are used for patients with HHS.

Which anatomic structure has influence over all areas of body functioning? a. Hypothalamus b. Pancreas c. Thyroid d. Pituitary

A The hypothalamus gland is known as the "master gland" because of the influence it has over all areas of body functioning. The hypothalamus controls pituitary gland action and response by secreting substances called release-inhibiting factors.

Where does the concentration and dilution of urine occur? a. In the juxtamedullary nephrons b. In the cortical nephrons c. In the peritubular capillaries d. In the internal nephron

A The juxtamedullary nephrons have long loops of Henle that have an important role in the concentration and dilution of urine. The peritubular capillaries, known as the vasa recta, surround the juxtamedullary nephrons, maintaining a concentration gradient to concentrate the urine. Most nephrons are cortical nephrons. Both types of cortical nephrons perform excretory and regulatory functions.

A patient is admitted with severe hypokalemia. On admission the patient's laboratory values are serum K, 2.2 mEq/L; blood urea nitrogen (BUN), 15 mg/dL; and creatinine, 1.2 mg/dL. Urine output is averaging 45 mL/h. The patient is given a total of 80 mEq of potassium over 4 hours. The potassium level is repeated and the result is K, 2.4 mEq/L. What other information would be beneficial at this time? a. Magnesium level b. Repeat creatinine level c. Calcium level d. Hemoglobin level

A The levels of other intracellular electrolytes, such as calcium and potassium, are affected by the level of magnesium. The most important functions of magnesium are ensuring the transport of sodium and potassium across the cell membrane and as a co-factor in many intracellular enzyme reactions. Depletion of magnesium liberates potassium to the extracellular fluid, which causes an increase in the excretion of potassium by the kidney and hypokalemia. If the patient has a low magnesium level, it could explain the lack of response to the potassium infusions. The other levels have little effect on serum potassium level or its response to infusions.

Which statement would indicate that a patient who has received an organ transplant understands the teaching about immunosuppressive medications? a. "My drug dosages will be lower because the medications enhance each other." b. "I will be less prone to side effects because I will be taking more than one drug." c. "Lower doses of these medications put me at greater risk for infection." d. "Taking more than one medication will put me at risk for developing allergies."

A These "triple-drug" regimens are designed to prevent rejection while reducing the toxicity of the individual medications.

A patient is admitted after a lung transplant. The nurse knows the patient is at risk for developing pneumonia. What parameter would be a priority for the nurse to monitor? a. Oxygen saturation b. Chest tube output c. Intake and output d. Blood pressure

A These patients are at risk for developing pneumonia. Aggressive postoperative pulmonary toilet is essential to promote airway clearance because surgical denervation of the lungs diminishes the cough reflex after surgery. Thus it is important to monitor oxygen saturation levels.

A patient who receives peritoneal dialysis is admitted after a 3-day history of flulike symptoms. The patient reports muscle cramps and is noted to have a low blood pressure and tachycardia. The nurse suspects the patient may be experiencing what condition? a. Dehydration b. Peritonitis c. Fluid obstruction d. Hernias

A This patient has dehydration. This patient is showing signs and symptoms of muscle cramps and low blood pressure.

A patient is admitted after surgery with a history of hyperthyroidism. The nurse suspects the patient may be developing thyroid storm. Which finding would confirm this suspicion? a. Tachycardia b. Hypotension c. Decreased appetite d. Hypothermia

A Thyroid storm, also called thyroid crisis, is a rare and life-threatening exacerbation of hyperthyroidism. The pathophysiology underlying the transition from hyperthyroidism to thyroid storm is not fully understood because thyroid hormone levels are not necessarily different from patients with hyperthyroidism. Atrial fibrillation is the most common dysrhythmia in patients with hyperthyroidism, and tachydysrhythmias should be anticipated in thyroid storm, especially in patients with underlying heart disease.

To remove fluid during hemodialysis, a positive hydrostatic pressure is applied to the blood and a negative hydrostatic pressure is applied to the dialysate bath. What is this process called? a. Ultrafiltration b. Hemodialysis c. Reverse osmosis d. Colloid extraction

A To remove fluid, a positive hydrostatic pressure is applied to the blood, and a negative hydrostatic pressure is applied to the dialysate bath. The two forces together, called transmembrane pressure, pull and squeeze the excess fluid from the blood. The difference between the two values (expressed in millimeters of mercury [mm Hg]) represents the transmembrane pressure and results in fluid extraction, known as ultrafiltration, from the vascular space.

The nurse is caring for a patient after a lung transplant. Which intervention would be a priority for the nurse? a. Wean the patient from the ventilator. b. Maintain hypotensive levels. c. Start corticosteroid therapy. d. Initiate pulmonary function studies.

A Ventilation settings are determined partially by the underlying disease process and patient progression. Regular suctioning of secretions is crucial to maintain airway clearance. Extubation is performed after satisfactory gas exchange and lung mechanics are accomplished, and most patients are extubated within 24 to 48 hours. Evaluation for graft dysfunction, reperfusion injury, gas trapping, and phrenic nerve injury is ongoing. Early mobilization after extubation is essential for improved pulmonary toilet and helps prevent reintubation.

A patient is brought to the hospital with a major stab wound. After excessive blood loss, the nurse should anticipate what change in the patient's serum osmolality? a. Increase b. Decrease c. Rapid decrease than increase d. Equal to his or her sodium level

A With hemorrhage the serum osmolality increases, which stimulates the release of antidiuretic hormone.

The neuroendocrine stress response produces which findings? (Select all that apply.) a. Elevated blood pressure b. Decreased gastric motility c. Tachycardia d. Heightened pain awareness e. Increased glucose

A, B, C, E The fight-or-flight response, or sympathetic nervous response, releases catecholamine that causes an increased heart rate and blood pressure. Blood is shunted from nonessential organs such as the stomach, glucose is made available to the brain cells, and pain awareness is decreased.

A patient is admitted with respiratory failure and is being mechanically ventilated. The nurse understands there is a significant association between acute kidney injury and respiratory failure. How does mechanical ventilation alter kidney function? (Select all that apply.) a. Decreases blood flow to the kidney b. Decreases glomerular filtration rate (GFR) c. Damages the kidney tubular endothelium d. Decreases urine output e. Hinders flow of urine from the kidneys

A, B, D Mechanical ventilation can alter kidney function. Positive-pressure ventilation reduces blood flow to the kidney, lowers the glomerular filtration rate (GFR), and decreases urine output. These effects are intensified with the addition of positive end-expiratory pressure (PEEP).

A patient is admitted in acute heart failure secondary to renal insufficiency. The patient reports that over the past few weeks, his urine output has decreased, and he has developed peripheral edema and ascites. A diagnosis of renal failure is made. The nurse would expect to see elevated values in which laboratory results? (Select all that apply.) a. Blood urea nitrogen (BUN) b. Creatinine c. Glucose d. Hemoglobin and hematocrit e. Protein

A, B, D With kidney dysfunction, the blood urea nitrogen (BUN) is elevated because of a decrease in the glomerular filtration rate and resulting decrease in urea excretion. Elevations in the BUN can be correlated with the clinical manifestations of uremia; as the BUN rises, symptoms of uremia become more pronounced. Creatinine levels are fairly constant and are affected by fewer factors than BUN. As a result, the serum creatinine level is a more sensitive and specific indicator of kidney function than BUN. Creatinine excess occurs most often in persons with kidney failure resulting from impaired excretion. Decreased hematocrit value can indicate fluid volume excess because of the dilutional effect of the extra fluid load. Decreases also can result from anemias, blood loss, liver damage, or hemolytic reactions. In individuals with acute kidney failure, anemia may occur early in the disease.

What causes the presence of myoglobin in urine? (Select all that apply.) a. Bleeding b. Traumatic damage to the skeletal muscle c. Asthmatic attack d. Rhabdomyolysis e. Cocaine abuse

A, B, D, E Although a few red blood cells (RBCs) in the urine are normal, discernibly bloody urine usually indicates bleeding within the urinary tract or kidney trauma. The presence of myoglobin can make the urine appear red. Microscopic examination of the urine fails to reveal RBCs, with myoglobin being present instead. Myoglobin in the urine may result from skeletal muscle damage (eg, traumatic crush injury) or rhabdomyolysis. Rhabdomyolysis may develop in patients admitted to a critical care unit for many reasons, including traumatic injury, cocaine abuse, status epilepticus, heat prostration, or collapse during intense physical exercise (eg, running a marathon race on a hot day).

A patient is admitted with Cushing syndrome. Which findings would the nurse expect to note as confirming this diagnosis? (Select all that apply.) a. Hirsutism b. Rounded face c. Hypotension d. Decreased libido e. Scleroderma f. Fatigue and weakness

A, B, D, F Signs and symptoms of Cushing syndrome include hypertension, thin skin that bruises easily, and poor wound healing.

What is the primary function of aldosterone? a. Excretion of potassium through the renal tubules b. Control of sodium and water c. Regulation of bicarbonate d. Reabsorption of sodium and potassium

B Aldosterone acts on the distal tubule to facilitate sodium and water resorption, resulting in an expanded circulating blood volume and increased blood pressure. When the arterial blood pressure increases, the juxtaglomerular apparatus reduces the release of renin, and the renin-angiotensin-aldosterone system is less active.

A patient was admitted with diabetic ketoacidosis (DKA). Glucose is 349 mg/dL, K is 3.7 mEq/L, and pH is 7.10. Which of the following interventions would you expect? (Select all that apply.) a. NS 1.5 L IV fluid bolus b. Insulin infusion at 5 units/h c. Sodium bicarbonate 50 mmol IV push d. Vasopressin 10 units IM every 3 hours e. Potassium 20 mEq/L of IV fluid

A, B, E Dehydration is a common presenting issue in diabetic ketoacidosis (DKA), so the administration of fluids and insulin will help correct the hyperglycemia and acidosis. Sodium bicarbonate is not recommended unless the pH is less than 6.9. As dehydration is reversed, potassium moves back into the cells, and hypokalemia can result, so administration of replacement potassium is necessary.

What are the functions of the kidneys? (Select all that apply.) a. Formation of urine b. Blood pressure regulation c. Erythrocyte destruction d. Breakdown of prostaglandins e. Regulation of acid-base balance

A, B, E The kidneys are complex organs responsible for numerous functions and substances necessary to maintain homeostasis. The primary roles of the kidneys are to remove metabolic wastes, maintain fluid and electrolyte balance, and help achieve acid-base balance. Hormones produced by the kidneys have an important role in blood pressure control, red blood cell production, and bone metabolism. The kidneys are important in maintaining the intracellular and extracellular environment required by all cells to function effectively.

The nurse is managing a patient with hyperglycemia. Which findings would the nurse expect to note to support this diagnosis? (Select all that apply.) a. Anorexia b. Abdominal pain c. Bradycardia d. Fluid overload e. Change in level of consciousness f. Kussmaul respirations

A, B, E, F More than likely the patient with hyperglycemia will be fluid volume depleted and tachycardic.

A patient with thyrotoxicosis is admitted. Which laboratory tests would the nurse expect to be ordered for this patient? (Select all that apply.) a. Total serum triiodothyronine (TT3) b. Total serum thyroxine (TT4) c. Free urine thyroid stimulating hormone d. Total urine thyroxine e. Thyroglobulin (Tg) f. Free thyroxine (T4)

A, B, E, F Thyroid tests include total serum thyroxine, free thyroxine, total serum triiodothyronine, free triiodothyronine, thyroid-stimulating hormone (thyrotropin), and thyroglobulin.

Potential recipients are matched with donors based on what factors? (Select all that apply.) a. Blood type b. Human leukocyte antigen c. Race d. Socioeconomic status e. Severity of illness f. Location of recipient g. Waiting time on the list

A, B, E, F, G Potential recipients are matched with the donor based on blood type, height, weight, human leukocyte antigen, distance from the donor, waiting time on the list, and severity of illness. The national system in place for organ allocation is fair and equitable for those requiring a transplant. The list does not reference race, gender, or socioeconomic status.

Which federal laws regulate the medical and surgical therapy involved with organ transplantation? (Select all that apply.) a. Omnibus Budget Reconciliation Act b. Uniform Anatomical Gift Act c. Hospital Conditions of Participation-Organ Donations d. Medical Examiner Laws e. Uniform Determination of Death Act

A, C The Uniform Anatomical Gift Act, Uniform Determination of Death Act, and Medical Examiner Laws Restricting Ability of Medical Examiner or Coroner to deny organ donation are all state laws.

A patient is being treated for hypothyroidism. The nurse knows that this disorder can affect a variety of physiologic processes. What are the major functions of the thyroid hormones? (Select all that apply.) a. Increase the rate of glucose used by the cells b. Decrease fat metabolism c. Increase the body's demand for vitamins d. Decrease cardiac output e. Stimulate bone resorption f. Increase serum cholesterol levels

A, C, E Major functions of thyroid hormones include interacting with growth hormone, maturation of the skeletal system, development of the central nervous system, stimulating carbohydrate metabolism, increasing the rate of glucose absorption from the gastrointestinal tract, increasing the rate of glucose use by the cells, accelerating the rate of fat metabolism, increasing cholesterol degradation in the liver, decreasing serum cholesterol levels, increasing protein anabolism and catabolism, mobilizing protein and releasing amino acids into circulation, increasing energy from protein nutrients through gluconeogenesis, increasing the body's demand for vitamins, increasing oxygen consumption and use, increasing basal metabolic rate, marked chronotropic and inotropic effects on the heart, increasing cardiac output, stimulating the contractility and excitability of the myocardium, increasing blood volume, expanding respiratory rate and depth necessary for normal hypoxic and hypercapnic drive, promoting sympathetic overactivity, boosting erythropoiesis, increasing metabolism and clearance of various hormones and pharmacologic agents, and stimulating bone resorption.

The nursing management plan for the patient with a urinary drainage catheter would include which interventions to prevent catheter-associated urinary tract infection (CAUTI)? (Select all that apply.) a. Insert urinary catheters using aseptic techniques. b. Change the urinary catheter daily. c. Review the need for the urinary catheter daily and remove promptly. d. Flush the urinary catheter q8 hours to maintain patency. e. Avoid unnecessary use of indwelling urinary catheters.

A, C, E The key components of catheter-associated urinary tract infection (CAUTI) prevention are to avoid unnecessary use of urinary catheters, insert urinary catheters using aseptic technique, adopt evidence-based standards for maintenance of urinary catheters, review the need for the urinary catheter daily, and remove the catheter promptly.

The nurse is caring for a patient who has had a recent heart transplant. Which signs and symptoms would alert the nurse that the patient is rejecting the transplant? (Select all that apply.) a. Shortness of breath b. Tolerance of exercise c. Disturbance in mood d. Decreased weight e. Pulmonary crackles f. Onset of hypertension g. Sudden onset of edema

A, C, E, G Signs and symptoms of heart transplant rejections include intolerance to exercise, increased weight, and onset of hypotension.

Which factors stimulate the release of antidiuretic hormone (ADH)? (Select all that apply.) a. Hypovolemia b. Hypernatremia c. Hypothermia d. Opioids e. Emotional stress

A, D, E Factors stimulating release of antidiuretic hormone include hyperosmolality of extracellular fluid, hypovolemia, increased body temperature, medications (opioids, antineoplastic, oral hypoglycemics, β-adrenergics), and severe emotional or physical stress.

A patient is admitted with a traumatic head injury. The patient starts to exhibit signs of a decrease in antidiuretic hormone (ADH). Which symptom would alert the nurse to this problem? a. Rapidly escalating hypertension b. Massive diuresis c. Pulmonary edema d. Hyperkalemia

B A low sodium level is associated with a low serum osmolality (hypoosmolar state). When sodium levels rise, plasma osmolality increases (hyperosmolar state). Antidiuretic hormone is then released to stimulate water resorption at the nephron to maintain sodium balance. This process decreases water loss from the body and subsequently concentrates and reduces urine volume. Fluid conserved in this manner is returned to the circulating plasma, where it dilutes the concentration (osmolality) of plasma.

A patient has been admitted with abdominal pain. The nurse notes that the patient's fasting blood glucose is 120 mg/dL. Which statement regarding this finding is accurate? a. This is a normal finding in critically ill patients. b. This finding is indicative of prediabetes, but another test should be done to confirm. c. This finding is lower than what the nurse would expect in a patient receiving intravenous fluids. d. This finding is indicative of diabetic ketoacidosis.

B A normal fasting plasma glucose (FPG) level is between 70 and 110 mg/dL. An FPG level between 110 and 126 mg/dL identifies a person who is prediabetic. An FPG level of greater than 126 mg/dL (7 mmol/L) is diagnostic of diabetes. In nonurgent settings, the test is repeated on another day to make sure the result is accurate.

A patient has been admitted with diabetic ketoacidosis (DKA). The nurse knows that the top priority in the initial treatment of DKA is which intervention? a. Lowering the blood sugar as quickly as possible b. Administering intravenous fluids c. Administering sodium bicarbonate d. Determining the precipitating cause

B A patient with diabetic ketoacidosis (DKA) is dehydrated and may have lost 5% to 10% of his or her body weight in fluids. A fluid deficit up to 6 L can exist in severe dehydration. Aggressive fluid replacement is provided to rehydrate both the intracellular and the extracellular compartments and prevent circulatory collapse.

Which finding is expected in the patient with hypothyroidism? a. Increased T4 b. Anemia c. Decreased thyroid stimulating hormone d. Hyperglycemia

B Anemia is a common problem that is present in 25% to 50% of patients with hypothyroidism. Symptoms of fatigue and depression are associated. Erythropoiesis (red blood cell production) is impaired and inadequate. Coagulation abnormalities may coexist.

A patient has been admitted with a brain tumor. The patient starts to exhibit signs of an increase in antidiuretic hormone (ADH). When large amounts of ADH are released, the nurse should anticipate what change in the patient's sodium levels? a. Increase b. Decrease c. Rapid increase than decrease d. Depends on potassium level

B Antidiuretic hormone (ADH) also contributes to control of the sodium level in the extracellular fluid by control of plasma osmolality. The sodium ion concentration in the plasma largely determines plasma osmolality. Osmoreceptors, located in the hypothalamus, are sensitive to changes in the circulating plasma osmolality. Because ADH causes the patient to retain free water, the patient will have a dilutional hyponatremia

A patient is admitted with a traumatic head injury. The patient starts to exhibit signs of a decrease in antidiuretic hormone (ADH). What is the function of ADH? a. Control blood pressure b. Regulate fluid balance c. Normalize potassium levels d. Maintain homeostasis

B Antidiuretic hormone has been identified as the single most important hormone responsible for regulating fluid balance within the body.

A patient was admitted with an infection that had to be treated with gentamicin, an aminoglycoside antibiotic. After 3 days of administration, the patient developed oliguria, and an elevated blood urea nitrogen and creatinine levels. The nurse suspects the patient has developed what type of kidney injury? a. Prerenal b. Intrarenal c. Anuric d. Postrenal

B Any condition that produces an ischemic or toxic insult directly at parenchymal nephron tissue places the patient at risk for development of intrarenal. Ischemic damage may be caused by prolonged hypotension or low cardiac output. Toxic injury reaction may occur in response to substances that damage the kidney tubular endothelium, such as some antimicrobial medications and the contrast dye used in radiologic diagnostic studies.

A patient was admitted with an infection that had to be treated with an aminoglycoside antibiotic. After a few days the patient developed oliguria and elevated blood urea nitrogen and creatinine levels. The patient's vital signs are stable. The nurse would anticipate the practitioner ordering which dialysis method for this patient? a. Peritoneal dialysis b. Hemodialysis c. Continuous renal replacement therapy d. Intermittent ultrafiltration

B As a treatment, hemodialysis separates and removes from the blood excess electrolytes, fluids, and toxins by means of a hemodialyzer. Hemodialysis would be the first choice for managing this patient with medication toxicity.

A patient has been admitted with uncontrolled atrial fibrillation and muscle wasting. The practitioner suspects the patient may have a thyroid disorder. The nurse auscultates a bruit over the thyroid. What does this finding indicate? a. Normal function b. Enlargement of the thyroid c. Hypoplasia of the thyroid d. Tumor of the thyroid

B Auscultation of the thyroid is accomplished by use of the bell portion of the stethoscope to identify a bruit or blowing noise from the circulation through the thyroid gland. The presence of a bruit indicates enlargement of the thyroid as evidenced by increased blood flow through the glandular tissue.

Chloride plays a major role in maintaining what physiologic process? a. Cellular immunity b. Serum osmolality c. Bone strength d. Adenosine triphosphate (ATP)

B Chloride plays a major role in maintaining serum osmolality, water balance, and acid-base balance. The primary function of phosphorus is the formation of adenosine triphosphate, which provides intracellular energy for active transport mechanisms across the cell membrane. Additional functions of phosphorus include cell membrane structure, acid-base balance, oxygen delivery to the tissues, cellular immunity, and bone strength.

A patient has been admitted with a brain mass. The practitioner suspects it might be a pituitary tumor and orders a computed tomography (CT) scan. What area of brain should be scanned to confirm this diagnosis? a. Frontal lobe b. Base of the skull c. Temporal lobe d. Anterior fossa

B Computed tomography (CT) of the base of the skull identifies pituitary tumors, blood clots, cysts, nodules, or other soft tissue masses.

The practitioner has ordered continuous renal replacement therapy (CRRT) for a patient with acute kidney injury. The patient needs both the removal of fluids and a moderate amount of solutes. Which type of CRRT would the nurse anticipate being started on this patient? a. Slow continuous ultrafiltration (SCUF) b. Continuous venovenous hemofiltration (CVVH) c. Continuous venovenous hemodialysis (CVVHD) d. Continuous venovenous hemodiafiltration (CVVHDF)

B Continuous venovenous hemofiltration (CVVH) is indicated when the patient's clinical condition warrants removal of significant volumes of fluid and solutes. Fluid is removed by ultrafiltration in volumes of 5 to 20 mL/min or up to 7 to 30 L/24 h. Removal of solutes such as urea, creatinine, and other small non-protein-bound toxins is accomplished by convection

The nurse is caring for a patient with a traumatic brain injury. Yesterday the patient weighed 62 kg and today the patient weighs 60 kg. How much fluid loss does this change in weight reflect? a. 1 L b. 2 L c. 4 L d. 10 L

B Daily weight changes coincide with fluid retention and fluid loss. 1 L of fluid lost or retained is equal to approximately 2.2 lb, or 1 kg, of weight gained or lost. This patient lost 2 kg of weight, which is equivalent to 2 L of fluid.

Decreased urine osmolality is a sign of which disorder? a. Hyperglycemia b. Diabetes insipidus c. Thyroid crisis d. Syndrome of inappropriate secretion of antidiuretic hormone

B Decreased urine osmolality is a sign of diabetes insipidus.

The nurse is caring for a patient with type 1 diabetes who was admitted with complaints of increased lethargy. Serum laboratory values validate the diagnosis of diabetic ketoacidosis (DKA). Which symptom is most suggestive of DKA? a. Irritability b. Excessive thirst c. Rapid weight gain d. Peripheral edema

B Diabetic ketoacidosis (DKA) has a predictable clinical presentation. It is usually preceded by patient complaints of malaise, headache, polyuria (excessive urination), polydipsia (excessive thirst), and polyphagia (excessive hunger). Nausea, vomiting, extreme fatigue, dehydration, and weight loss follow. Central nervous system depression, with changes in the level of consciousness, can lead quickly to coma.

A patient has been admitted with uncontrolled atrial fibrillation and muscle wasting. The practitioner suspects the patient may have a thyroid disorder. The nurse anticipates the practitioner will initially order which diagnostic procedure to visualize the thyroid? a. Magnetic resonance imaging (MRI) b. Ultrasound c. Biopsy d. Computed tomography (CT)

B Diagnostic tests often begin with ultrasonography to visualize a thyroid nodule or tumor.

A patient is receiving total parenteral nutrition (TPN). During rounds the dietitian states that she believes the patient is getting too much glucose. Identify one area in which excess glucose is stored as glycogen. a. Pancreas b. Hepatic cells c. Gallbladder d. Stomach

B Excess glucose in the form of glycogen is stored in the hepatic and muscle cells for use as fuel at a later time.

The patient has been admitted from a skilled nursing facility with urinary sepsis. The patient has severe muscle wasting. What is the process by which fat and protein are converted to glucose for fuel? a. Glycogenolysis b. Gluconeogenesis c. Digestion d. Biotransformation

B For long-term energy needs, glucagon stimulates glucose release through the more complex process of gluconeogenesis. In gluconeogenesis, fat and protein nutrients are rapidly broken down into end products that are then changed into glucose.

A patient has thyroid storm. The nurse is providing medication instruction for home. The patient asks, "If I have a fever, should I take Tylenol or aspirin?" Which response would be the most appropriate? a. "Either one is fine because they do not affect the antithyroid medication." b. "Take Tylenol rather than aspirin because aspirin increases the amount of free thyroid hormone in circulation." c. "Take aspirin rather than Tylenol because Tylenol increases the amount of free thyroid hormone in circulation." d. "They both prevent the antithyroid medication from working correctly. I would recommend an NSAID."

B For management or elevated temperature, patients are instructed to use acetaminophen rather than salicylates because salicylates increase the amount of free thyroid hormone in circulation.

When a patient with diabetic ketoacidosis (DKA) has insulin infusing intravenously, the nurse expects a drop in the serum levels of which electrolytes? a. Sodium and potassium b. Potassium and phosphate c. Bicarbonate and calcium d. Sodium and phosphate

B Frequent verification of the serum potassium concentration is required for patients with DKA receiving fluid resuscitation and insulin therapy. The serum phosphate level is sometimes low (hypophosphatemia) in DKA. Insulin treatment may make this more obvious as phosphate is returned to the interior of the cell. If the serum phosphate level is less than 1 mg/dL, phosphate replacement is recommended.

A patient has been admitted post thyroidectomy. The nurse knows that adequate thyroid function is dependent on which factor? a. Basal metabolic rate b. Dietary intake of iodine c. Colloid osmotic pressure d. Sodium levels in the blood

B Functioning of the thyroid gland depends on many factors that respond to a delicate hormonal interplay; the hypothalamus, anterior pituitary, dietary intake of iodine, and circulating protein bodies in the blood all affect thyroid gland function.

Which statement made by a patient who has received an organ transplant indicates that the teaching was effective? a. "I will finally be able to eat a regular diet." b. "I will establish a routine for checking any skin changes." c. "I will check my blood pressure regularly to make sure it's not too high." d. "After my drug regimen is established, I won't have to worry about rejection."

B Immunosuppressed patients are at greater risk for developing skin cancer. After transplantation, such a patient needs to monitor for hypotension and eat a diet that is low in sodium, fat, and cholesterol.

A patient is admitted in acute heart failure secondary to renal insufficiency. The patient reports that over the past few weeks, his urine output has decreased, and he has developed peripheral edema and ascites. Which diagnostic tests would provide the best information about the internal kidney structures, such as the parenchyma, calyces, pelvis, ureters, and bladder? a. Kidney-ureter-bladder (KUB) b. Intravenous pyelography (IVP) c. Renal ultrasonography (ECHO) d. Renal angiography

B Intravenous pyelography allows visualization of the internal kidney parenchyma, calyces, pelvis, ureters, and bladder. Kidney-ureter-bladder flat-plate radiography of the abdomen determines the position, size, and structure of the kidneys, urinary tract, and pelvis. It is useful for evaluating the presence of calculi and masses and is usually followed by additional tests. In ultrasonography, high-frequency sound waves are transmitted to the kidneys and urinary tract, and the image is viewed on an oscilloscope. This noninvasive procedure identifies fluid accumulation or obstruction, cysts, stones or calculi, and masses. It is useful for evaluating the kidneys before biopsy. Angiography is injection of contrast into arterial blood perfusing the kidneys. It allows for visualization of renal blood flow and may also visualize stenosis, cysts, clots, trauma, and infarctions.

A patient with diabetic ketoacidosis (DKA) has an insulin drip infusing, and the nurse has just administered subcutaneous insulin. The nurse is alert for signs of hypoglycemia, which would include what findings? a. Kussmaul respirations and flushed skin b. Irritability and paresthesia c. Abdominal cramps and nausea d. Hypotension and itching

B Irritability and paresthesia are seen in hypoglycemia.

A patient is admitted with severe hyperglycemia due to new-onset type 1 diabetes mellitus. The nurse notes a sweet-smelling odor on the patient's exhaled breath. What causes this phenomenon? a. Metabolic alkalosis b. Ketoacidosis c. Glycosylation d. Dehydration

B Ketoacidosis results in the patient's breathing becoming deep and rapid (Kussmaul respirations) and the patient's breath having a fruity odor. Metabolic alkalosis and dehydration do not cause this phenomenon. Glycosylation is when the hemoglobin within each cell binds to the available blood glucose.

Which medication is classified as a loop diuretic? a. Acetazolamide b. Furosemide c. Mannitol d. Metolazone

B Loop diuretics include furosemide, bumetanide, and torsemide. Furosemide is the most frequently used diuretic in critical care patients. It may be administered orally, as an intravenous (IV) bolus, or as a continuous IV infusion. Diamox is a carbonic anhydrase inhibitor diuretic. Mannitol is an osmotic diuretic, and metolazone is a thiazide diuretic.

A patient reports losing weight even though she eats "everything in sight." She also reports tremors and diarrhea. The nurse suspects the patient may have what problem? a. Hypothyroidism b. Diabetes mellitus c. Hyperthyroidism d. Pituitary tumor

C Weight loss, increased appetite, tremors, insomnia, and diarrhea are symptoms of hyperthyroidism.

Stimulation from what receptors will cause contraction of the detrusor muscle of the bladder? a. Bladder wall and ureters b. Bladder wall and urethra c. Ureters and urethra d. Urethra and bladder neck

B Nervous system control in the urinary tract is reflected in the process of micturition, or the release of urine. Bladder fullness stimulates stretch receptors in the bladder wall and a portion of the urethra. Signals are carried through nerves in the sacral area and return as parasympathetic messages to contract the detrusor muscle of the bladder.

A patient has been on complete bed rest for 3 days. The practitioner has left orders to get the patient out of bed for meals. The patient complains of feeling dizzy and faint while sitting at the bedside. The nurse suspects that the patient is experiencing what problem? a. Orthostatic hypertension b. Orthostatic hypotension c. Hypervolemia d. Electrolyte imbalance

B Orthostatic hypotension produces subjective feelings of weakness, dizziness, or faintness. Orthostatic hypotension occurs with hypovolemia or prolonged bed rest or as a side effect of medications that affect blood volume or blood pressure.

Patients discharged with antithyroid medications should be alerted to which potential side effect? a. Hyperthermia b. Agranulocytosis c. Tachypnea d. Diaphoresis

B Patients discharged with antithyroid medications are alerted to the potential side effect of agranulocytosis. Symptoms of agranulocytosis include sudden cough, fever, rash, and inflammation.

Which pathophysiologic mechanism occurs in the syndrome of inappropriate antidiuretic hormone (SIADH)? a. Massive diuresis, leading to hemoconcentration b. Dilutional hyponatremia, reducing sodium concentration to critically low levels c. Hypokalemia from massive diuresis d. Serum osmolality greater than 350 mOsm/kg

B Patients with the syndrome of inappropriate antidiuretic hormone (SIADH) have an excess of antidiuretic hormone secreted into the bloodstream, more than the amount needed to maintain normal blood volume and serum osmolality. Excessive water is resorbed at the kidney tubule, leading to dilutional hyponatremia.

Percussion of kidneys is usually done to assess what parameter? a. Size and shape of the kidneys b. Presence of pain in the renal area c. Presence of a fluid wave d. Patient's overall fluid status

B Percussion is performed to detect pain in the area of a kidney or to determine excess accumulation of air, fluid, or solids around the kidneys. Percussion of the kidneys also provides information about kidney location, size, and possible problems.

A patient is admitted with diabetic ketoacidosis (DKA) and is experiencing polyuria. Which electrolyte disturbances would the nurse expect to see at this phase of DKA? a. Decreased calcium and increased phosphorus levels b. Decreased potassium and sodium levels. c. Increased sodium and decreased phosphorus levels d. Decreased calcium and potassium levels.

B Polyuria results in large volumes of water, along with sodium, potassium, and phosphorus, being excreted in the urine, causing a fluid volume deficit.

A patient has developed acute kidney injury (AKI) secondary to hemorrhage shock. Which intravenous solution would the nurse expect to be ordered for this patient? a. Dextrose in water b. Normal saline c. Albumin d. Lactated Ringer solution

B Prerenal acute kidney injury (AKI) is caused by decreased perfusion and flow to the kidney. It is often associated with trauma, hemorrhage, hypotension, and major fluid losses. If contrast dye is used, aggressive fluid resuscitation with normal saline is recommended.

A patient has developed acute kidney injury (AKI) secondary to cardiogenic shock. Which laboratory value would the nurse find helpful in evaluating patient's renal status? a. Serum sodium b. Serum creatinine c. Serum potassium d. Urine potassium

B Serum creatinine is the most reliable predictor of kidney function. In the acutely ill patient, small changes in the serum creatinine level and urine output may signal important declines in the glomerular filtration rate and kidney function.

A patient has been diagnosed with syndrome of inappropriate secretion of antidiuretic hormone (SIADH). The nurse is administering hypertonic saline. The nurse knows that the serum sodium should not be raised more than how many milliequivalents per day? a. 5 mEq/day b. 10 mEq/day c. 20 mEq/day d. 25 mEq/day

B Serum sodium levels must be evaluated at least every 4 hours during the acute phase of sodium replacement. The serum sodium should not be raised more than 10 mEq/L in 24 hours.

A patient was admitted with acute kidney failure. Which urinalysis value reflects a decreased ability of the kidneys to concentrate urine? a. pH of 5.0 b. Specific gravity of 1.000 c. No casts d. Urine sodium of 140 mEq/24 h

B Specific gravity measures the density or weight of urine compared with that of distilled water. The normal urinary specific gravity is 1.005 to 1.025. For comparison, the specific gravity of distilled water is 1.000. Because urine is composed of many solutes and substances suspended in water, the specific gravity should always be higher than that of water.

A patient was admitted with acute heart failure a few days ago. Today the patient's urine has a specific gravity of 1.040. What could be the potential cause for this value? a. Volume overload b. Volume deficit c. Acidosis d. Urine ketones

B Specific gravity ranges from 1.003 to 1.030. Possible causes for increased values include volume deficit, glycosuria, proteinuria, and prerenal acute kidney injury (AKI). Possible causes for decreased values include volume overload and interrenal AKI.

What is the most common site for short-term vascular access for immediate hemodialysis? a. Subclavian artery b. Subclavian vein c. Femoral artery d. Radial vein

B Subclavian and femoral veins are catheterized when short-term access is required or when a graft or fistula vascular access is nonfunctional in a patient requiring immediate hemodialysis. Subclavian and femoral catheters are routinely inserted at the bedside. Most temporary catheters are venous lines only. Blood flows out toward the dialyzer and flows back to the patient through the same catheterized vein. A dual-lumen venous catheter is most commonly used.

What hormone does the adrenal cortex secrete to maintain homeostasis? a. Androgens b. Aldosterone c. Adrenocorticotrophic hormone (ACTH) d. Corticosteroid

B The adrenal cortex secretes cortisol. It regulates fluid homeostasis by means of aldosterone, and it secretes androgens.

A patient is admitted post arrest from sudden cardiac death. According the code record the patient received a total of 6 mg of epinephrine. The nurse knows that epinephrine and norepinephrine are also secreted by which organ? a. Adrenal cortex b. Adrenal medulla c. Anterior pituitary d. Posterior pituitary

B The adrenal medulla (inner region) secretes two important catecholamines: epinephrine, also known as adrenaline, and norepinephrine, also known as noradrenaline.

The nurse has admitted a patient with hyperglycemic hyperosmolar state (HHS). Which findings would the nurse expect to observe in this patient? a. Hyperglycemia with low serum osmolality b. Severe hyperglycemia with minimal or absent ketosis c. Little or no ketosis in serum with rapidly escalating ketonuria d. Hyperglycemia and ketosis

B The hallmarks of HHS are extremely high levels of plasma glucose with resulting elevations in serum hyperosmolality and osmotic diuresis. The disorder occurs mainly in patients with type 2 diabetes.

The mobilization of calcium from bone stores is accomplished through the influence of which hormone? a. Antidiuretic hormone (ADH) b. Parathyroid hormone (PTH) c. Thyroid-stimulating hormone (TSH) d. Erythropoietin

B The mobilization of calcium from bone stores is accomplished through the influence of parathyroid hormone. The kidneys secrete erythropoietin, the hormone that controls erythrocyte (red blood cell) production in the bone marrow.

A patient is admitted with syndrome of inappropriate secretion of antidiuretic hormone (SIADH). The nurse understands that the hyponatremia associated with this disorder is the result of which mechanism? a. Increased cortisol release b. Excessive water reabsorption c. Excessive sodium excretion d. Decreased glucagon release

B The patient with syndrome of inappropriate secretion of antidiuretic hormone (SIADH) has an excess of ADH secreted into the bloodstream, more than the amount needed to maintain normal blood volume and serum osmolality. Excessive water is resorbed at the kidney tubule, leading to dilutional hyponatremia.

An alert and oriented patient presents with a pulmonary artery occlusion pressure (PAOP) of 4 mm Hg, blood pressure of 88/50 mm Hg, cardiac index of 1.8, and urine output of 15 mL/h. The patient's blood urea nitrogen (BUN) is 44 mg/dL and creatinine is 3.2 mg/dL. Lungs are clear to auscultation with no peripheral edema noted. Which treatment would the nurse expect the practitioner to order? a. Lasix 40 mg intravenous push b. 0.9% normal saline at 125 mL/h c. Dopamine 15 mcg/kg/min d. Transfuse 1 U of packed red blood cells

B The patient's hemodynamic parameters are most consistent with hypovolemia. The acute kidney injury would probably be prerenal from inadequate blood flow. The treatment of choice for hypovolemia is fluid resuscitation. Important criteria when calculating fluid volume replacement include baseline metabolism, environmental temperature, and humidity. The rate of replacement depends on cardiopulmonary reserve, adequacy of kidney function, urine output, fluid balance, ongoing loss, and type of fluid replaced.

An older patient presents with a serum glucose level of 900 mg/dL, hematocrit of 55%, and no serum ketones. Immediate attention must be given to which intervention? a. Evaluating clotting factors b. Fluid administration c. Insulin administration d. Sodium replacement

B The physical examination may reveal a profound fluid deficit. Signs of severe dehydration include longitudinal wrinkles in the tongue, decreased salivation, and decreased central venous pressure, with increases in heart rate and rapid respirations (Kussmaul air hunger does not occur). In older patients, assessment of clinical signs of dehydration is challenging.

What is the recommended nutritional intake of protein to control azotemia in the patient with acute kidney injury? a. 0.5 to 1.0 g/kg/day b. 1.2 to 1.5 g/kg/day c. 1.7 to 2.5 g/kg/day d. 2.5 to 3.5 g/kg/day

B The recommended energy intake is between 20 and 30 kcal/kg/day, with 1.2 to 1.5 g/kg of protein per day to control azotemia (increased blood urea nitrogen level).

The following substances, among others, are found in a patient's urine sample: urea, creatinine, sodium, chlorine, potassium, glucose, and bicarbonate ions. Which patient situation could account for this abnormal finding? a. Blood pressure of 76/30 mm Hg b. Blood glucose of 456 mg/dL c. Blood glucose of 40 mg/dL d. Blood potassium level of 4.1 mEq/L

B This glucose reading is above the threshold concentration. Glucose is normally completely reabsorbed from the tubules. Above the threshold level, the tubules are unable to reabsorb all of the glucose, and some spills into the urine. All of the other findings in this urine sample are normal findings.

A patient is admitted with hyponatremia. The practitioner suspects the patient may have syndrome of inappropriate secretion of antidiuretic hormone (SIADH) and orders a serum ADH level for the next morning. Which medications must be stopped or withheld at least 8 hours prior to the test? a. Insulin and furosemide b. Morphine and carbamazepine c. Digoxin and potassium d. Heparin and lopressor

B To prepare the patient for the test, all drugs that may alter the release of antidiuretic hormone (ADH) are withheld for a minimum of 8 hours. Common medications that affect ADH levels include morphine sulfate, lithium carbonate, chlorothiazide, carbamazepine, oxytocin, nicotine, alcohol, and selective serotonin reuptake inhibitors.

Which pathophysiologic mechanism occurs in the patient with type 2 diabetes? a. Lack of insulin production b. Imbalance between insulin production and use c. Overproduction of glucose d. Increased uptake of glucose in the cells

B Type 2 diabetes results from a progressive insulin secretory defect in addition to insulin resistance.

When the brain perceives a stressful or threatening situation, the hypothalamus releases ________________________, which causes the anterior pituitary to release______________. a. cortisol; androgens b. corticotropin-releasing hormone; adrenocorticotrophic hormone (ACTH) c. ACTH; glucocorticoid hormone d. ACTH; epinephrine

B When the brain perceives a stressful or threatening situation, the hypothalamus releases corticotropin-releasing hormone, which acts on the anterior pituitary to release adrenocorticotrophic hormone, which circulates in the bloodstream to reach the adrenal cortex and stimulate glucocorticoid hormone release.

The nurse knows that during the resuscitation of a patient with diabetic ketoacidosis (DKA), the intravenous (IV) line should be changed to a Patient becomes more alert b. IV insulin has been infusing for 4 hours c. Blood glucose drops to 200 mg/dL d. Blood glucose drops to 100 mg/dL

C When the serum glucose level decreases to 200 mg/dL, the infusing solution is changed to a 50/50 mix of hypotonic saline and 5% dextrose. Dextrose is added to replenish depleted cellular glucose as the circulating serum glucose level falls.

A patient has been admitted in acute heart failure. Which parameter would indicate to the nurse that the patient is fluid overloaded? a. Central venous pressure of 4 mm Hg b. Pulmonary artery occlusion pressure (PAOP) of 18 mm Hg c. Cardiac index of 2.5 L/min/m2 d. Mean arterial pressure of 40 mm Hg

B pulmonary artery occlusion pressure (PAOP) represents the left atrial pressure required to fill the left ventricle. When the left ventricle is full at the end of diastole, this represents the volume of blood available for ejection. It is also known as left ventricular preload and is measured by the PAOP. The normal PAOP is 5 to 12 mm Hg. In fluid volume excess, PAOP rises. In fluid volume deficit, PAOP is low.

The organ procurement organization coordinator writes orders to initiate standard donor care. Which interventions are parts of standard donor care? (Select all that apply.) a. Continue administration of osmotic agents and diuretics. b. Maintain head of bed at 30 to 40 degrees elevation. c. Continue routine pulmonary suctioning and side-to-side body positioning. d. Warming blanket to maintain body temperature above 36.5° C. e. Maintain mean arterial pressure less than 70 mm Hg.

B, C, D Standard donor care includes: maintain head of bed at 30 to 40 degrees elevation, continue routine pulmonary suctioning and side-to-side body positioning, and warming blanket to maintain body temperature above 36.5° C. Osmotic agents and diuretics should be discontinued. Mean arterial pressure should be maintained above 70 mm Hg.

Many barriers exist to increasing the number of organ donors to meet today's growing need. Which barriers are related to hospital factors? (Select all that apply.) a. Limited resources b. Lack of interest c. Failure to approve protocols d. Nurses' attitudes e. Delay in termination of life support

B, C, D, E A recent study of hospital donation practices and their impact on organ donation outcomes revealed gaps in knowledge of organ donation; brain death; referral criteria; and at times, a poor relationship between the hospital and the organ procurement organization. It is important that nurses are knowledgeable about the organ donation process. Nurses must assess their own beliefs that pertain to organ donation because the attitude of the nurse and care given to the family can impact the outcome of the donation.

Not every patient with end-stage liver disease is a candidate for receiving a transplant. Which conditions are contraindications to a kidney transplant? (Select all that apply.) a. Malignancy during the past 5 years b. Active infectious process c. Advanced cardiopulmonary disease d. Recreational drug use e. Nonadherence to current medical regimen

B, C, D, E Contraindications to kidney transplantation include malignancy during the past 3 years, active infectious process, advanced cardiopulmonary disease, high risk for surgery, nonadherence to current medical regimen, recreational drug use, and other serious contributing disease processes.

A patient with bronchogenic oat cell carcinoma has a drop in urine output. The laboratory reports a serum sodium level of 120 mEq/L, a serum osmolality level of 220 mOsm/kg, and urine specific gravity of 1.035. The nurse suspects the patient may be developing what problem? a. Diabetes ketoacidosis (DKA) b. Diabetes insipidus (DI) c. Syndrome of inappropriate secretion of antidiuretic hormone (SIADH) d. Hyperaldosteronism

C A decreased urine output, hyponatremia, hypoosmolality, and high urine specific gravity are classic signs of syndrome of inappropriate secretion of antidiuretic hormone (SIADH). Oat cell carcinoma is a precipitating factor for SIADH.

A patient was admitted with acute heart failure who has been receiving diuretic therapy. The nurse suspects the patient is hypovolemic. What auscultatory parameter would confirm the nurse's suspicion? a. Hypertension b. Third or fourth heart sound c. Orthostatic hypotension d. Vascular bruit

C A drop in systolic blood pressure of 20 mm Hg or more, a drop in diastolic blood pressure of 10 mm Hg or more, or a rise in pulse rate of more than 15 beats/min from lying to sitting or from sitting to standing indicates orthostatic hypotension. The drop in blood pressure occurs because a sufficient preload is not immediately available when the patient changes position. The heart rate increases in an attempt to maintain cardiac output and circulation.

A nurse is caring for a patient who is having difficulty maintaining a normal blood sugar level. In the healthy body, how is a normal blood glucose level is maintained? a. Insulin secretion b. Cellular metabolism c. Insulin-to-glucagon ratio d. Insulin-to-thyroxine ratio

C A normal blood glucose level is maintained by the insulin-to-glucagon ratio. When the blood glucose level is high, insulin is released and glucagon is inhibited. When blood glucose levels are low, glucagon rather than insulin is released.

A patient in diabetic ketoacidosis (DKA) is comatose with a temperature of 102.2° F. The nurse suspects the patient may have which secondary disorder? a. Head injury b. Hypothalamus infarction c. Infection d. Heat stroke

C A patient in diabetic ketoacidosis can experience a variety of complications, including fluid volume overload, hypoglycemia, hypokalemia or hyperkalemia, hyponatremia, cerebral edema, and infection.

The nurse is caring for a patient after a liver transplant. Which finding would be of most concern to the nurse? a. Increased serum glucose b. Low aspartate aminotransferase (AST) and alanine aminotransferase (ALT) c. Elevated prothrombin time d. Decreased blood urea nitrogen and creatinine

C A patient with suspected primary nonfunction of a liver graft demonstrates (1) hemodynamic instability, (2) progressive deterioration of kidney function, (3) coagulopathies and abnormal serum liver function laboratory test results, (4) hypoglycemia, (5) continued ventilatory dependence, and (6) an inability to awaken from anesthesia.

Which type of intravenous fluid will not create a shift of fluids within the vascular space? a. Hypertonic b. Hypotonic c. Isotonic d. Osmotic pressure

C An isotonic solution has roughly the same concentration of particles as the blood plasma; cells within an isotonic solution maintain consistency and do not lose or gain fluid to their surroundings. A hypertonic solution contains a greater concentration of particles than that inside the cell and causes fluid to be drawn out of the cells. Used inappropriately, too much fluid may be withdrawn, causing a withering of the cell (crenation). A hypotonic solution contains a lesser concentration of particles than that inside the cell and causes fluid to be drawn into the cells. If used incorrectly, a hypotonic solution can cause too much fluid to enter the cell, causing the cells to swell and burst (hemolysis). Osmotic pressure is created by solutes and other substances (eg, albumin, globulin, fibrinogen) suspended in fluid.

An elderly patient is in a motor vehicle accident and sustains a significant internal hemorrhage. The nurse knows the patient is at risk for developing what type of acute kidney injury (AKI)? a. Intrinsic b. Postrenal c. Prerenal d. Intrarenal

C Any condition that decreases blood flow, blood pressure, or kidney perfusion before arterial blood reaches the renal artery that supplies the kidney may be anatomically described as prerenal acute kidney injury (AKI). When arterial hypoperfusion caused by low cardiac output, hemorrhage, vasodilation, thrombosis, or other cause reduces the blood flow to the kidney, glomerular filtration decreases, and consequently urine output decreases. Any condition that produces an ischemic or toxic insult directly at parenchymal nephron tissue places the patient at risk for development of intrarenal AKI. Any obstruction that hinders the flow of urine from beyond the kidney through the remainder of the urinary tract may lead to postrenal AKI.

Patients who have sustained head trauma or have undergone resection of a pituitary tumor have an increased risk of developing which disorder? a. Type 1 diabetes b. Thyrotoxicosis c. Diabetes insipidus d. Myxedema coma

C Any patient who has head trauma or resection of a pituitary tumor has an increased risk of developing diabetes insipidus.

To prevent acid-base disturbances, what is the ratio between carbonic acid and bicarbonate? a. 10 mEq of carbonic acid to 20 mEq of bicarbonate b. 20 mEq of carbonic acid to 10 mEq of bicarbonate c. 1 mEq of carbonic acid to 20 mEq of bicarbonate d. 20 mEq of carbonic acid to 1 mEq of bicarbonate

C Bicarbonate levels in the body are in balance with carbonic acid (H2CO3) levels. The ratio between the two must remain proportional at 1 mEq of carbonic acid to 20 mEq of bicarbonate; otherwise, acid-base disturbances will result.

A patient is admitted with massive head trauma. The patient is unresponsive and on mechanical ventilation. The electroencephalogram (EEG) is negative for brain waves. The family has agreed to organ donation. Which organs would not be considered for transplant? a. Heart b. Lung c. Liver d. Kidney

C Brain death is an absolute contraindication for a liver transplant.

A patient with acute kidney injury (AKI) has been started on continuous venovenous hemodiafiltration (CVVHDF). The nurse understands the patient should be closely monitored for what circuit-related complications of the therapy? a. Hypervolemia, hypothermia, and hyperkalemia b. Access dislodgment, decreased outflow pressures, and bleeding c. Filter clotting, access failure, and air embolism d. Increased overflow pressure, dehydration, and calcium loss

C Circuit-related complications of continuous renal replacement therapy include air embolism, clotted hemofilter, poor ultrafiltration, blood leaks, broken filter, disconnection, access failure, and catheter dislodgement

A patient has been admitted with multiple trauma due to a motor vehicle accident several weeks ago. Given the prolonged critical illness, the nurse knows the patient may no longer secrete adequate amounts of what hormone? a. Aldosterone b. Adrenocorticotropic hormone (ACTH) c. Cortisol d. Antidiuretic hormone (ADH)

C Cortisol is secreted in response to physiologic stress as a result of infection, trauma, and hypoglycemia. Early in critical illness, a rise in cortisol levels can be documented. However, over time the adrenal gland may not be able to secrete adequate amounts of stress hormones, especially when critical illness is prolonged. Aldosterone is secreted in response to intravascular hypovolemia.

The nurse is teaching a patient about antiviral medications. The patient asks what is the most common viral infection in transplant recipients. What should the nurse tell the patient? a. Influenza b. Respiratory syncytial virus c. Cytomegalovirus (CMV) d. Parainfluenza

C Cytomegalovirus (CMV) is the most common viral infection in transplant recipients. CMV may occur within the first few months after transplantation and may occur later on. Other common viruses after transplant include respiratory syncytial virus, influenza, adenovirus, human metapneumovirus, and parainfluenza. These are not treated prophylactically but as they occur.

What is the functional unit of the kidney called? a. Bowman capsule b. Glomerulus c. Nephron d. Distal tubule

C Each kidney is made up of about 1 million nephrons, the functional units of the kidneys. Each nephron is made up of several distinct structures, which are the glomerulus, Bowman capsule, proximal tubule, loop of Henle, distal tubule, and collecting duct.

Glucagon release is stimulated by which physiologic factor? a. Increases in circulating insulin b. Decreases in blood amino acids c. Stimulation of the sympathetic nervous system d. High blood glucose levels

C Glucagon release from the pancreas is stimulated by low blood glucose levels, starvation, exercise, or stimulation of the sympathetic nervous system.

The nurse is caring for a patient with myxedema coma. The patient's temperature is 93° F. Which intervention would the nurse include in the plan of care for this patient? a. Give aggressive therapy that includes warm peritoneal lavage. b. Allow the patient to maintain this body temperature to decrease oxygen demands. c. Use warming blankets to slowly warm the patient. d. Wait until the patient shivers to start warming.

C Hypothermia will gradually improve as the patient is treated with thyroid hormone. Several warm blankets comfortably wrapped around the patient (with mild hypothermia) may be sufficient to help raise the body temperature to normal. Active warming devices are also used. Continuous assessments are important to avoid too-rapid heating and vasodilation.

A patient with chronic kidney disease receives hemodialysis treatments 3 days a week. Every 2 weeks, the patient requires a transfusion of 1 or 2 U of packed red blood cells. What is the probable reason for this patient's frequent transfusion needs? a. Too much blood phlebotomized for tests b. Increased destruction of red blood cells because of the increased toxin levels c. Lack of production of erythropoietin to stimulate red blood cell formation d. Hemodilution secondary to fluid retention

C In chronic kidney disease, the kidneys do not produce sufficient amounts of erythropoietin in response to normal stimuli such as anemia or hypotension. The other choices are not reasons for frequent blood transfusions in this patient.

A patient with thyrotoxicosis has been admitted. The nurse knows that when levels of T3 and T4 are elevated, the pituitary gland inhibits what hormone? a. Thyroglobulin b. Thyroid-releasing hormone (TRH) c. Thyroid-stimulating hormone (TSH) d. Calcitonin

C In response to decreased circulating levels of T3 and T4, the hypothalamus releases thyroid-releasing hormone (TRH). TRH activates thyroid-stimulating hormone (TSH) in the anterior pituitary, and TSH stimulates the thyroid gland to manufacture and release the thyroid hormones T3 and T4 in the presence of iodine.

A patient is admitted in acute heart failure secondary to renal insufficiency. The patient reports that over the past few weeks, his urine output has decreased, and he has developed peripheral edema and ascites. The nurse suspects the main cause of ascites is what condition? a. Hypervolemia b. Dehydration c. Volume overload d. Liver damage

C Individuals with kidney failure may have ascites caused by volume overload, which forces fluid into the abdomen because of increased capillary hydrostatic pressures. However, ascites may or may not represent fluid volume excess. Severe ascites in persons with compromised liver function may result from decreased plasma proteins. The ascites occurs because the increased vascular pressure associated with liver dysfunction forces fluid and plasma proteins from the vascular space into the interstitial space and abdominal cavity. Although the individual may exhibit marked edema, the intravascular space is volume depleted, and the patient is hypovolemic.

A patient is receiving insulin due to the development of steroid-induced hyperglycemia. In addition to lowering blood glucose levels, what other physiologic effect may occur when insulin is administered? a. Breakdown of proteins b. Hypercalcemia c. Hypokalemia d. Cellular dehydration

C Insulin is a potent anabolic hormone that produces hypoglycemia and augments the transport of potassium into the cells. With potassium driven into the cells, serum potassium may decrease, resulting in hypokalemia.

The nurse is caring for a patient who has been newly diagnosed with type 1 diabetes. Which laboratory results would the nurse note confirming this diagnosis? a. Hemoglobin A1C of 3% b. Absence of ketones in the urine c. Presence of ketones in the blood d. Fasting glucose of 105 mg/dL

C Ketone bodies are a byproduct of rapid fat breakdown. Ketone blood levels rise in acute illness, fasting, and with sustained elevation of blood glucose in type 1 diabetes in the absence of insulin. The patient would also have ketones in the urine, a hemoglobin A1C greater than 6%, and a fasting glucose greater than 125 mg/dL.

A patient was admitted with liver failure and acute kidney injury (AKI). Which intravenous solution should the nurse question if it were ordered for this patient? a. D5W b. 0.9% NaCl c. Lactated Ringer solution d. 0.45% NaCl

C Lactated Ringer solution is contraindicated for patients with kidney or liver diseases or in lactic acidosis.

What is the dose for low-dose dopamine? a. 1 to 2 mcg/kg/min b. 1 to 2 mg/kg/min c. 2 to 3 mcg/kg/min d. 2 to 3 mg/kg/min

C Low-dose dopamine (2 to 3 mcg/kg/min), previously known as renal-dose dopamine, is frequently infused to stimulate blood flow to the kidney. Dopamine is effective in increasing urine output in the short term, but tolerance of the dopamine renal receptor to the medication is theorized to develop in the critically ill patients who are most at risk for acute kidney injury.

The nurse is caring for a critically ill patient with type 1 diabetes. The nurse understands that the patient is at risk for developing diabetic ketoacidosis (DKA) secondary to what etiology? a. Excess insulin administration b. Inadequate food intake c. Physiologic and psychologic stress d. Increased release of antidiuretic hormone (ADH)

C Major neurologic and endocrine changes occur when an individual is confronted with physiologic stress caused by any critical illness, sepsis, trauma, major surgery, or underlying cardiovascular disease.

A patient is admitted with massive head trauma. The patient is unresponsive and on mechanical ventilation. The electroencephalogram (EEG) is negative for brain waves. The family has agreed to organ donation. When the nurse turns the patient's head quickly from side to side, the patient's eyes do not move. The nurse knows this is the result of the loss of what reflex? a. Oculovestibular b. Corneal c. Oculocephalic d. Pupillary

C Oculocephalic reflex: Ocular movements are lost with brain death. The oculocephalic reflex, also described as doll's eyes, involves fast turning of the head to both sides. In brain death, this should not generate any eye movements. Oculovestibular reflex: Because the oculovestibular reflex is tested using iced water or normal saline, it is sometimes called cold calorics. The head of the bed is elevated 30 degrees, and approximately 50 mL of ice water or normal saline is injected into the ear; no movement of the eye toward the side of the stimulus should be present. It is recommended that the patient be observed for up to 1 minute after each ear irrigation, and 5 minutes should be allowed before testing the opposite ear. Corneal and jaw reflexes: Facial sensory and motor responses are elicited by testing for corneal and jaw reflexes. Stroking a cotton-tipped swab gently across the cornea tests the corneal reflexes. Pupillary reflexes: Pupillary signs are evaluated by absence of the light reflex, which is consistent with brain death. Most often the pupils are round, oval, or irregularly shaped, although dilated pupils may remain even after brain death has occurred.

A patient is admitted with renal failure. The patient's weight upon admission was 176 lb, and the next day it is 184 lb. What is the approximate amount of fluid retained with this weight gain? a. 800 mL b. 2200 mL c. 3600 mL d. 8000 mL

C One liter of fluid equals 1 kg, which is 2.2 lb; 8 lb equals 3.6 kg, which is 3.6 L; 3.6 L is equal to 3600 mL.

A patient with acute kidney injury (AKI) has been started on continuous venovenous hemodialysis (CVVHD). The nurse understands the patient should be closely monitored for what patient-related complications of the therapy? a. Air embolism, access failure, and blood leaks b. Decreased inflow pressure, air bubbles, and power surge c. Infection, hypotension, and electrolyte imbalances d. Catheter dislodgement, decreased outflow pressure, and acid-base imbalances

C Patient-related complications of continuous renal replacement therapy (CRRT) include dehydration, hypotension, electrolyte imbalances, acid-base imbalances, blood loss, hemorrhage, hypotension, and infection.

The nursing management plan for a patient with thyrotoxicosis would include which intervention? a. Providing diversional stimuli b. Restricting fluids c. Maintaining a quiet, restful environment d. Administering thyroid supplements at the same time each day

C Patients in thyroid storm are agitated, anxious, and unable to rest, and they benefit from an environment that is calm. Gradually, the effects of the antithyroid medications, iodides, and -adrenergic blocking drugs will decrease the neurologic symptoms related to the catecholamine sensitivity. Frequent reassurance and clear, simple explanations of the patient's condition help decrease the fear brought on by the onset thyroid storm

Maintaining a normal range of serum potassium is important for what physiologic process? a. Fluid regulation b. Acid-base buffering c. Nervous impulse conduction d. Triggering antidiuretic hormone (ADH) release

C Potassium functions in the body to aid in nervous impulse conduction and muscle contraction.

A patient is admitted with a long history of mental illness. The patient's spouse states the patient has been drinking up to 10 gallons of water each day for the past 2 days and refuses to eat. The patient is severely dehydrated and soaked with urine. The nurse suspects the patient has which problem? a. Central diabetes insipidus (DI) b. Nephrogenic DI c. Dipsogenic DI d. Iatrogenic DI

C Psychogenic diabetes insipidus (DI) is a rare form of the disease that occurs with compulsive drinking of more than 5 L of water a day. Long-standing psychogenic DI closely mimics nephrogenic DI because the kidney tubules become less responsive to antidiuretic hormone as a result of prolonged conditioning to hypotonic urine.

A patient is admitted with diabetic ketoacidosis (DKA). The patient's arterial blood gas indicates the patient has an uncompensated metabolic acidosis. The patient has rapid, regular respirations. Which medical intervention would the nurse expect to initiate to correct the acidosis? a. Initiate oxygen therapy via a face mask. b. Administer sodium bicarbonate. c. Administer insulin and fluids intravenously. d. Prepare for intubation.

C Replacement of fluid volume and insulin interrupts the ketotic cycle and reverses the metabolic acidosis. In the presence of insulin, glucose enters the cells, and the body ceases to convert fats into glucose.

A nurse is initiating a patient teaching plan for a patient with hypothyroidism. The patient is currently taking salicylates, lithium, and sulfonamides. It is important that the nurse tell the patient that these medications influence thyroid levels. How are thyroid levels affected by these medications? a. They increase T3 b. They increasing T4 c. They decrease T3 d. They decrease T4

C Salicylates, lithium, and sulfonamides can cause a decrease in T3 levels.

Which statement best describes the effects of somatostatin? a. It stimulates the release of antidiuretic hormone and oxytocin. b. It suppresses the release of pancreatic polypeptide. c. It inhibits the release of insulin and glucagon. d. It inhibits the release of thyroid-stimulating hormone and thyroid hormone.

C Somatostatin decreases glucagon secretion, and in high quantities, it decreases insulin release. Hyperglycemia stimulates the activity of the delta cells. It is theorized that the release of insulin enables somatostatin to control beta-cell activity. Somatostatin may be involved in the regulation of the postprandial influx of glucose into cells.

What happens to patients that meet criteria for imminent death? a. They are placed on life support and referred to the organ procurement organizations. b. They are referred to the organ procurement organizations at the time of their death. c. They are referred to the organ procurement organizations in a timely manner. d. They are referred to a practitioner to make the decision if they are candidates for organ donation.

C The Centers for Medicare and Medicaid Services guidelines, the Joint Commission standards, and hospital policies require that patients meeting criteria for imminent death and cardiac death be referred to an organ procurement organization in a timely manner.

A patient has been admitted after surgery for removal of a brain tumor. The nurse suspects the patient may be developing diabetes insipidus (DI). Which findings would confirm the nurse's suspicion? a. Hyperglycemia and hyperosmolarity b. Hyperglycemia and peripheral edema c. Intense thirst and passage of excessively large quantities of dilute urine d. Peripheral edema and pulmonary crackles

C The clinical diagnosis is made by the dramatic increase in dilute urine output in the absence of diuretics, a fluid challenge, or hyperglycemia. Characteristics of DI are intense thirst and the passage of excessively large quantities of very dilute urine.

Through what mechanism does the endocrine system help maintain equilibrium? a. Control of smooth muscle b. Control of skeletal muscle c. Hormones d. Neuronal control

C The endocrine system controls and communicates by distributing potent hormones throughout the body.

The initial filtering of the blood occurs in which structure? a. The distal tubule b. The proximal tubule c. The glomerulus d. The collecting tubule

C The first structure of each nephron is the glomerulus, a high-pressure capillary bed that serves as the filtering point for the blood. Positive filtration pressure in the glomerulus is achieved as a result of the high arterial pressure as the blood enters the afferent arteriole and the resistance created by the smaller efferent arteriole as the blood exits the glomerulus. As a result of the positive-pressure gradient, fluid and solutes are filtered through the glomerular capillary walls.

A patient with hyperglycemic hyperosmolar state (HHS) has a serum glucose level of 400 mg/dL and a serum sodium level of 138 mEq/L. What is the intravenous fluid of choice? a. D5W c. 0.9% NS b. 0.45% NS d. D5/NS

C The fluid deficit may be as much as 150 mL/kg of body weight. The average 150-lb adult can lose more than 7 to 10 L of fluid. Physiologic saline solution (0.9%) is infused at 1 L/h, especially for patients in hypovolemic shock if there is no cardiovascular contraindication. Several liters of volume replacement may be required to achieve a blood pressure and central venous pressure within normal range. Infusion volumes are adjusted according to the patient's hydration state and sodium level.

To assess whether or not an arteriovenous fistula is functioning, what must the nurse do and why? a. Palpate the quality of the pulse distal to the site to determine whether a thrill is present; auscultate with a stethoscope to appreciate a bruit to assess the quality of the blood flow. b. Palpate the quality of the pulse proximal to the site to determine whether a thrill is present; auscultate with a stethoscope to appreciate a bruit to assess the quality of the blood flow. c. Palpate gently over the site of the fistula to determine whether a thrill is present; listen with a stethoscope over this site to appreciate a bruit to assess the quality of the blood flow. d. Palpate over the site of the fistula to determine whether a thrill is present; check whether the extremity is pink and warm.

C The nurse frequently assesses the quality of blood flow through the fistula. A patent fistula has a thrill when palpated gently with the fingers and a bruit when auscultated with a stethoscope. The extremity should be pink and warm to the touch. No blood pressure measurements, intravenous infusions, or laboratory phlebotomy is performed on the arm with the fistula.

Which nursing intervention should be initiated on all patients with the syndrome of inappropriate antidiuretic hormone (SIADH)? a. Placing the patient on an air mattress b. Forcing fluids c. Initiating seizure precautions d. Applying soft restraints

C The patient with the syndrome of inappropriate antidiuretic hormone (SIADH) has an excess of ADH secreted into the bloodstream, more than the amount needed to maintain normal blood volume and serum osmolality. Excessive water is resorbed at the kidney tubule, leading to dilutional hyponatremia. Symptoms of severe hyponatremia include an inability to concentrate, mental confusion, apprehension, seizures, a decreased level of consciousness, coma, and death.

A patient was admitted with heart failure. The nurse is assessing the patient for peripheral edema. The nurse presses two fingers over the tibial area, and it takes 1 minute before the indention disappears. What would the nurse note in the medical record? a. +1 pitting edema b. +2 pitting edema c. +3 pitting edema d. +4 pitting edema

C The pitting edema scale includes +1 = 2-mm depth; +2 = 4-mm depth (lasting up to 15 seconds); +3 = 6-mm depth (lasting up to 60 seconds); and +4 = 8-mm depth (lasting longer than 60 seconds).

The nurse understands that the onset of seizures in the patient with diabetes insipidus (DI) is indicative of which situation? a. Increased potassium levels b. Hyperosmolality c. Severe dehydration d. Toxic ammonia levels

C This excessive intake of water reduces the serum osmolality to a more normal level and prevents dehydration. In the person with a decreased level of consciousness, the polyuria leads to severe hypernatremia, dehydration, decreased cerebral perfusion, seizures, loss of consciousness, and death.

A patient weighs 140 kg and is 60 inches tall. The patient's blood sugar is being controlled by glipizide. Which topic would be important for the nurse to include in the patient's discharge education plan? a. Signs of hyperglycemia b. Proper injection technique c. Weight loss d. Increased caloric intake

C This patient weighs 308 lb and is 5 feet tall. Diet management and exercise are interventions to facilitate weight loss in patients with type 2 diabetes.

A patient has been admitted with acute kidney injury. Which serum laboratory values would the nurse expect to be ordered to confirm this diagnosis? a. Sodium and potassium b. Creatinine and calcium c. Blood urea nitrogen (BUN) and creatine d. Potassium and magnesium

C Urea and creatinine are the primary waste products that are measured in determining kidney function. Urea is measured as blood urea nitrogen and is the end product of protein metabolism and results from the breakdown of ammonia in the liver. Like urea, creatinine accumulates when the glomerulus is unable to filter it from the blood.

A patient is admitted with hypernatremia secondary to neurogenic diabetes insipidus. The nurse notes that the patient's serum osmolality is 350 mOsm/kg. What does this finding indicate? a. The patient is overhydrated. b. The patient's serum osmolality is normal. c. The patient is dehydrated. d. The patient is hypothyroid.

C Values for serum osmolality in the bloodstream range from 275 to 295 mOsm/kg H2O. Increased serum osmolality stimulates the release of antidiuretic hormone, which in turn reduces the amount of water lost through the kidney.

As serum osmolality rises, intravascular fluid equilibrium will be maintained by the release of what substance? a. Ketones b. Glucagon c. Antidiuretic hormone d. Potassium

C When the serum osmolality level increases, antidiuretic hormone is released from the posterior pituitary gland and stimulates increased water resorption in the kidney tubules. This expands the vascular space, returns the serum osmolality level back to normal, and results in more concentrated urine and an elevated urine osmolality level.

A patient is admitted with critical hypotension, hyperkalemia, hyponatremia, and hypoglycemia. The nurse knows that these symptoms are highly suggestive of which disorder? a. Myxedema b. Diabetes insipidus c. Addisonian crisis d. Cushing syndrome

C disonian crisis is a life-threatening condition in which the adrenal gland is almost nonfunctional, usually because of destruction of adrenal tissue. The patient presents acutely with critical hypotension, an elevated serum potassium level (hyperkalemia), a low serum sodium level (hyponatremia), and hypoglycemia.

The nurse is caring for a patient with diabetes insipidus, which is the result of a decrease in antidiuretic hormone (ADH). Which statements regarding the stimulation or inhibition of ADH are accurate? (Select all that apply.) a. Infection stimulates the release of ADH. b. Cold stimulates the release of ADH. c. Emesis stimulates the release of ADH. d. Opioids inhibit the release of ADH. e. Hemorrhage inhibits the release of ADH. f. Pain inhibits the release of ADH.

C, D Emesis, hemorrhage, and pain stimulate the release of antidiuretic hormone (ADH). Cold and opioids inhibit the release of ADH. Infection does not directly affect the release of ADH.

One therapeutic measure for treating hyperkalemia is the administration of dextrose and regular insulin. Which statement regarding how this treatment works is accurate? a. Glucose and insulin force potassium out of the cells, lowering it on a cellular level. b. Glucose and insulin promote higher excretion of potassium in the urine. c. Glucose and insulin bind with potassium, lowering available amounts. d. Glucose and insulin force potassium into the cells, lowering it on a serum level.

D Acute hyperkalemia can be treated temporarily by intravenous administration of insulin and glucose. An infusion of 50 mL of 50% dextrose accompanied by 10 units of regular insulin forces potassium out of the serum and into the cells.

Which electrolytes are cations? a. Sodium, potassium, and chloride b. Sodium, chloride, and bicarbonate c. Bicarbonate, chloride, and calcium d. Sodium, potassium, and magnesium

D A balance exists between cations (positively charged ions), anions (negatively charged ions), and other substances in the fluid compartments. Cations are sodium, potassium, magnesium, and calcium. Anions are chloride and bicarbonate.

The patient complains of a metallic taste and loss of appetite. The nurse is concerned that the patient has developed what problem? a. Glycosuria b. Proteinuria c. Myoglobin d. Uremia

D A history of recent onset of nausea and vomiting or appetite loss caused by taste changes (uremia often causes a metallic taste) may provide clues to the rapid onset of kidney problems. Glycosuria is the presence of glucose in the urine. Proteinuria is the presence of protein in the urine. Myoglobin is the presence of red blood cells in the urine.

Which medication can place a patient at risk for developing syndrome of inappropriate secretion of antidiuretic hormone (SIADH)? a. Adenosine b. Diltiazem c. Heparin sodium d. Acetaminophen

D Acetaminophen increases the release of antidiuretic hormone.

A patient is admitted with severe hyperglycemia. The patient is very lethargic and has a "fruity" odor to his breath. The nurse knows the odor on the patient's breath is indicative of which situation? a. Alcohol intoxication b. Lack of sodium bicarbonate c. Hypokalemia d. Presence of acetone

D Acid ketones dissociate and yield hydrogen ions (H+) that accumulate and precipitate a decrease in serum pH. The level of serum bicarbonate also decreases, consistent with a diagnosis of metabolic acidosis. Breathing becomes deep and rapid (Kussmaul respirations) to release carbonic acid in the form of carbon dioxide. Acetone is exhaled, giving the breath its characteristic "fruity" odor.

A patient with a history of type 2 diabetes was admitted after aneurysm repair. The patient's serum glucose levels have been elevated for the past 2 days, and the patient is concerned about becoming dependent on insulin. Which statement is the nurse's best response to the patient's concerns? a. "This surgery may have damaged your pancreas. We will have to do more evaluation." b. "Perhaps your diabetes was more serious from the beginning." c. "You will need to discuss this with your physician." d. "The stress on your body has temporarily increased your blood sugar levels."

D Adrenal hormones released during stress elevate blood sugar by increasing insulin resistance and increasing hepatic gluconeogenesis.

In caring for a patient with a thyrotoxicosis, the nurse would expect to observe which neurologic symptom? a. Lethargy b. Depression c. Seizures d. Agitation

D Agitation and the inability to rest are often found in a patient with thyrotoxic crisis.

A patient has been admitted after a craniotomy. The patient starts to exhibit changes in the level of antidiuretic hormone (ADH). The nurse knows that ADH works primarily affects which organ? a. Liver b. Pancreas c. Stomach d. Kidneys

D Antidiuretic hormone has two functions: (1) By means of the V1 receptors, it constricts smooth muscles within the arterial wall, and (2) through V2 receptors, it regulates fluid balance by altering the permeability of the kidney tubule to water.

A patient was admitted with multiple trauma who has been volume resuscitated. The nurse suspects the patient is fluid overloaded. Which assessment findings would confirm the nurse's suspicion? a. Venous filling of the hand veins greater than 5 seconds b. Distended neck veins in the supine position c. Presence of orthostatic hypotension d. Presence of a third heart sound

D Auscultation of the heart requires not only assessing rate and rhythm but also listening for extra sounds. Fluid overload is often accompanied by a third or fourth heart sound, which is best heard with the bell of the stethoscope.

A patient is reporting a headache, fatigue, abdominal pain, and blurred vision. The nurse knows that these signs may indicate the patient has what problem? a. Hypothyroidism b. Pituitary tumor c. Cushing syndrome d. Hyperglycemia

D Because severe hyperglycemia affects a variety of body systems, all systems are assessed. The patient may complain of blurred vision, headache, weakness, fatigue, drowsiness, anorexia, nausea, and abdominal pain.

A patient with acute kidney injury (AKI) has been started on continuous venovenous hemodialysis (CVVHD). The nurse understands that this type of continuous renal replacement therapy (CRRT) is indicated for the patient who needs what type of treatment? a. Fluid removal only b. Fluid removal and moderate solute removal c. Fluid removal and maximum solute removal d. Maximum fluid and solute removal

D Continuous venovenous hemodialysis (CVVHD) is indicated for patients who require large-volume removal of fluid and solutes.

A patient is admitted with multiple trauma. Which hormone would the nurse expect to be increased in response to physiologic stress? a. Mineralocorticoid b. Corticosteroid c. Glucocorticoid d. Cortisol

D Cortisol is released in response to physiologic stress caused by infection, trauma, and the fasting state.

A patient has been admitted with acute kidney injury. Which parameter would the nurse expect to find to confirm this diagnosis? a. Decreased blood urea nitrogen b. Bibasilar lung crackles c. Peripheral edema d. Decreased creatinine clearance

D Creatinine is used as a measure of the glomerular filtration rate (GFR) because it is a waste product produced at a fairly constant rate by the muscles, is freely filtered by the glomerulus, and is minimally resorbed or secreted by the tubules. Therefore, most of the creatinine produced by the body is excreted by the kidneys, making the creatinine clearance a good screening and follow-up test for estimating the GFR. A creatinine clearance less than 100 mL/min reflects a GFR of less than 100 mL/min and is a signal of decreased kidney function. A creatinine clearance (and GFR) less than 20 mL/min results in symptoms of kidney failure.

In a patient with a distended abdomen, differentiating ascites from solid bowel contents is accomplished by performing what assessment? a. Auscultation of bowel sounds b. Palpation of the liver margin c. Measuring abdominal girth d. Eliciting a fluid wave

D Differentiating ascites from distortion by solid bowel contents is accomplished by producing what is called a fluid wave. The fluid wave is elicited by exerting pressure to the abdominal midline while one hand is placed on the right or left flank. Tapping the opposite flank produces a wave in the accumulated fluid that can be felt under the hands.

Rejection that occurs within hours after the transplantation and results in immediate graft failure is referred to as what type of rejection? a. Acute b. Intermediate c. Chronic d. Hyperacute

D Hyperacute rejection is a humoral-mediated response, which occurs within hours after transplantation and results in immediate graft failure. Acute rejection tends to occur weeks to months after transplantation but can occur at any time. Chronic rejection occurs at varying times after transplantation and progresses for years until it leads to ultimate failure of the transplanted organ.

A patient was admitted with diabetic ketoacidosis (DKA) an hour ago and is on an insulin drip. Suddenly, the nurse notices frequent premature ventricular contractions (PVCs) on the electrocardiogram. The nurse notifies the practitioner. The nurse would anticipate an order for which intervention? a. A lidocaine bolus b. Stopping the insulin drip c. Synchronizing cardioversion d. Obtaining serum electrolytes

D Hypokalemia can occur within the first hours of rehydration and insulin treatment. Continuous cardiac monitoring is required, because low serum potassium (hypokalemia) can cause ventricular dysrhythmias.

A patient is being evaluated for thyroid dysfunction. The laboratory findings include a decreased serum free thyroxine (T4) level and an elevated thyroid-stimulating hormone (TSH) level. These findings confirm which diagnosis? a. Cushing syndrome b. Addison disease c. Thyrotoxicosis d. Hypothyroidism

D Hypothyroidism is indicated by a high thyroid-stimulating hormone (TSH) and low serum T4 levels. Thyrotoxicosis is hyperthyroidism and is indicated by very low TSH, high serum T4, and increased T3:T4 ratio. Addison disease is a rare disorder of the adrenal cortex that involves hyposecretion of glucocorticoids (cortisol), sometimes occurring with hyposecretion of mineralocorticoids (aldosterone). Cushing syndrome is caused by the excess release of the glucocorticoid hormone cortisol.

Ammonia, hydrogen, and ammonium are secreted in what part of the kidney? a. Loop of Henle b. Collecting duct c. Glomerulus d. Proximal tubule

D In addition to its major role in resorbing water and solutes from the filtrate, the proximal tubule secretes organic anions and cations into the tubular lumen. Ammonia is produced from the metabolism of glutamine in the mitochondria of the proximal tubule cells, where ammonia (NH3) combines with hydrogen (H) to form ammonium (NH), which is secreted into the proximal tubule lumen.

A patient is admitted with hypernatremia secondary to diabetes insipidus (DI). The practitioner suspects the patient has neurogenic DI. Which finding would confirm that diagnosis? a. A slight increase in urine osmolality b. A decrease in urine output c. A decrease in serum osmolality d. No change in urine osmolality

D In cases of severe central diabetes insipidus (DI)d the urine osmolality shows a significant increase (becomes more concentrated). Test results in which urine osmolality remains unchanged indicate nephrogenic DI.

A patient with type 2 diabetes is admitted. He is very lethargic and hypotensive. A diagnosis of hyperglycemic hyperosmolar syndrome (HHS) is given. Which findings support this diagnosis? a. Decreased serum glucose and increased serum ketones b. Increased urine ketones and decreased serum osmolality c. Increased serum osmolality and increased serum potassium d. Increased serum osmolality and increased serum glucose

D Laboratory findings are used to establish the definitive diagnosis of hyperglycemic hyperosmolar syndrome (HHS). Plasma glucose levels are strikingly elevated (greater than 600 mg/dL). Serum osmolality is greater than 320 mOsm/kg.

A patient has been admitted with a severe kidney infection. The nurse suspects the patient has damage to the glomerular membrane. Which substance in the urine would confirm the nurse's suspicion? a. Creatinine b. Bicarbonate ions c. Sodium d. Albumin

D Large molecules such as albumin and red blood cells are prevented from entering the filtrate. The presence of large molecules in the urine is a signal that the glomerular membrane is damaged or affected by disease.

A patient with liver failure manifested by portal hypertension and ascites is hospitalized and waiting for a liver transplant. Which order should the nurse question? a. Low-sodium diet b. Low-protein diet c. ABO typing d. Human leukocyte antigen tissue typing

D Liver failure is managed with a low-protein diet to decrease the risk of hepatic encephalopathy. A low-sodium diet is ordered to help manage the ascites. Blood type and body size are used to match a liver donor, not human leukocyte antigen tissue testing.

The nurse is caring for a patient with type 1 diabetes who was admitted with complaints of increased lethargy. Serum laboratory values validate the diagnosis of diabetic ketoacidosis (DKA). Which statement best describes the rationale for administrating potassium supplements with the patient's insulin therapy? a. Potassium replaces losses incurred with diuresis. b. The patient has been in a long-term malnourished state. c. Intravenous (IV) potassium renders the infused solution isotonic. d. Insulin drives the potassium back into the cells.

D Low serum potassium (hypokalemia) occurs as insulin promotes the return of potassium into the cell and metabolic acidosis is reversed. Replacement of potassium by administration of potassium chloride (KCl) begins as soon as the serum potassium falls below normal. Frequent verification of the serum potassium concentration is required for patients with diabetic ketoacidosis (DKA) who are receiving fluid resuscitation and insulin therapy.

Which electrolyte abnormality is evident early in the course of kidney failure? a. Sodium b. Potassium c. Chloride d. Phosphorus

D Phosphorus abnormalities are evident early in the course of kidney failure.

Which condition would result in an increased release of renin? a. Increased release of angiotensin I b. Increased release of angiotensin II c. Increased amount of sodium in the distal convoluted tubule d. Reduced pressure in the glomerulus

D Renin is released in response to reduced pressure in the glomerulus, sympathetic stimulation of the kidneys, and a decrease in the amount of sodium in the distal convoluted tubule. Renin is converted to angiotensin I, which is converted to the powerful vasoconstrictor angiotensin II. Angiotensin II stimulates the adrenal glands to secrete aldosterone, which acts on the distal tubules to resorb sodium from the tubular lumen into the circulation. When sodium is retained, so is water. Angiotensin II also constricts the renal vasculature, reducing kidney blood flow and available glomerular filtrate, sending a signal to the posterior pituitary to release antidiuretic hormone. The two systems intertwine to maintain fluid and electrolyte balance.

The nurse is caring for a patient with the syndrome of inappropriate secretion of antidiuretic hormone (SIADH). Which findings would confirm this diagnosis? a. Decreased ADH level and hyperkalemia b. Decreased ADH level and hypernatremia c. Increased ADH level and serum ketones d. Increased ADH level and low serum osmolality

D Secretion of antidiuretic hormone (SIADH) occurs when there are increased levels of ADH in the blood compared with a low serum osmolality.

The nurse is caring for a patient with a traumatic brain injury. The nurse suspects the patient is developing diabetes insipidus. Which test or procedure would confirm this diagnosis? a. Skull radiographs b. Serum glucose level c. Water deprivation test d. Antidiuretic hormone (ADH) stimulation test

D Serum antidiuretic hormone (ADH) levels are compared with the blood and urine osmolality to differentiate syndrome of inappropriate antidiuretic hormone (SIADH) from central diabetes insipidus (DI). Increased ADH levels in the bloodstream compared with a low serum osmolality and elevated urine osmolality confirms the diagnosis of SIADH. Reduced levels of serum ADH in a patient with high serum osmolality, hypernatremia, and reduced urine concentration signal central DI.

A patient has been admitted who is suspected of having thyrotoxicosis. Which symptom would support this diagnosis? a. Lethargy despite adequate sleep b. Bradycardia not related to medications c. Constipation despite laxatives d. Weight loss despite increased appetite

D Signs and symptoms of thyrotoxicosis include tremors, insomnia, increased appetite, diarrhea, muscle weakness or wasting, and a change in menstruation.

A patient is admitted with extreme fatigue, vomiting, and headache. This patient has type 1 diabetes that has been well controlled with an insulin pump. The patient states, "I know it could not be my diabetes because my pump gives me 24-hour control." Which reply would be the nurse's best response? a. "You know a lot about your pump, and you are correct." b. "You're right. This is probably a virus." c. "We'll get an abdominal CT and see if your pancreas is inflamed." d. "We'll check your serum blood glucose and ketones."

D Subcutaneous insulin pumps can malfunction. It is critical to assess glucose and ketone levels to evaluate for diabetic ketoacidosis.

A patient is admitted with hypernatremia secondary to diabetes insipidus (DI). Which test would help the practitioner differentiate between central and nephrogenic DI? a. Water deprivation test b. Serum osmolality c. Serum antidiuretic hormone level d. Antidiuretic hormone (ADH) test

D The antidiuretic hormone test is used to differentiate between neurogenic diabetes insipidus (DI) (central) and nephrogenic (kidney) DI. In severe central DI, in which the pituitary is affected, the urine osmolality shows a significant increase (becomes more concentrated), which indicates that the cell receptor sites on the kidney tubules are responsive to vasopressin. Test results in which urine osmolality remains unchanged indicate nephrogenic DI, suggesting kidney dysfunction because the kidneys are no longer responsive to antidiuretic hormone.

A patient is admitted with sepsis and acute kidney injury (AKI). The patient is started on continuous renal replacement therapy (CRRT). The nurse knows that fluid that is removed each hour is charted as what on the CRRT flowsheet? a. Convection b. Diffusion c. Replacement fluid d. Ultrafiltrate

D The fluid that is removed each hour is not called urine; it is known as ultrafiltrate. Typically, some of the ultrafiltrate is replaced through the continuous renal replacement therapy circuit by a sterile replacement fluid. Diffusion is the movement of solutes along a concentration gradient from a high concentration to a low concentration across a semipermeable membrane. Convection occurs when a pressure gradient is set up so that the water is pushed or pumped across the dialysis filter and carries the solutes from the bloodstream with it.

A patient is admitted with severe hyperglycemia due to new-onset type 1 diabetes mellitus. Which signs and symptoms obtained as part of the patient's history might indicate the presence of hyperglycemia? a. Recent episodes of tachycardia and missed heart beats b. Decreased urine output accompanied by peripheral edema c. Periods of hyperactivity with weight gain d. Increased thirst and increased urinary output

D The patient or family member may relay information about recent, unexplained changes in weight, thirst, hunger, and urination patterns.

The release of hormones by the pituitary gland is under the control which part of the brain? a. Cerebellum b. Occipital lobe c. Temporal lobe d. Hypothalamus

D The pituitary gland communicates with the hypothalamus by means of a vascular network.

A patient is admitted with diabetic ketoacidosis (DKA). The nurse knows that the lack of insulin results in which process? a. Decreased glucagon release b. Decreased glycogenolysis c. Decreased ketone production d. Increased gluconeogenesis

D The release of glucagon from the liver is stimulated when insulin is ineffective in providing the cells with glucose for energy. Glucagon increases the amount of glucose in the bloodstream by breaking down stored glucose (glycogenolysis). In insulin deficiency states, fat is rapidly converted into glucose (gluconeogenesis). Ketoacidosis occurs when free fatty acids are metabolized into ketones: Acetoacetate, â-hydroxybutyrate, and acetone are the three ketone bodies that are produced.

A patient is admitted with a massive head trauma. The patient is unresponsive and on mechanical ventilation. The electroencephalogram is negative for brain waves. The family has agreed to organ donation. How would the nurse check for the absence of cerebral motor reflexes? a. Performing a sternal rub b. Applying needle pricks to fingers and toes c. Clapping hands near the ears d. Applying pressure to the nail beds or supraorbital ridge

D These motor responses can be stimulated by the application of pressure to the nail beds or supraorbital ridge.

A patient who has received a transplant is being taught about cyclosporine. Which statement made by the patient would indicate the teaching was effective? a. "I know this drug prevents my immune system from working." b. "If I find the capsules are hard to swallow, I'll take the liquid." c. "I will need to watch for bruising." d. "I will need to monitor my blood pressure."

D This drug can cause hypertension. Capsules and liquid form are not interchangeable. The immune system still has some ability to work. Cyclosporine does not affect the bone marrow.

Laboratory results come back on a newly admitted patient: Serum blood urea nitrogen, 64 mg/dL; serum creatinine, 2.4 mg/dL; urine osmolality, 210 mOsm/kg; specific gravity, 1.002; and urine sodium, 96 mEq/L. The patient's urine output has been 120 mL since admission 2 hours ago. These values are most consistent with which diagnosis? a. Prerenal acute kidney injury b. Postrenal acute kidney injury c. Oliguric acute kidney injury d. Intrarenal acute kidney injury

D Urinary sodium less than 10 mEq/L (low) suggests a prerenal condition. Urinary sodium greater than 40 mEq/L (in the presence of an elevated serum creatinine and the absence of a high salt load) suggests intrarenal damage has occurred. The urine output does not seem to suggest oliguria. The other options do not fit the data as presented.

When a patient's blood pressure drops, which physiologic mechanism helps maintain adequate glomerular blood flow? a. Constriction of the afferent arteriole b. Dilation of the efferent arteriole c. Dilation of the collecting tubule d. Constriction of the efferent arteriole

D When the mean arterial blood pressure is decreased, the afferent arteriole dilates and the efferent arteriole constricts to maintain a higher pressure in the glomerular capillary bed and maintain the glomerular filtration rate at 125 mL/min. The ability of the kidneys to autoregulate blood flow begins to fail when the mean arterial blood pressure is less than 80 mm Hg or greater than 180 mm Hg.


Set pelajaran terkait

Security Operations and Monitoring

View Set

Chapter 9 Planning for Community Change

View Set

Restorative Exam 3 (with dental casting alloys cards)

View Set

GIS: GEOCODING AND DYNAMIC SEGMENTATION chp 16

View Set

Marketing Chap 13, Personal selling and Sales Promotions

View Set

Newton's Laws of Motion Examples

View Set

LS 1 Week 4 Chapter 13 Fluid and Electrolytes: Balance and Disturbance

View Set

Chapter 1 Test Review (AP Classroom)

View Set